Download as docx, pdf, or txt
Download as docx, pdf, or txt
You are on page 1of 69

Chapter III: LAW OF PATENTS

A. PATENTABLE SUBJECT MATTER:

1. Diamond v. Chakravarty:

FACTS:

Anand Mohan Chakravarty, filed a patent application, assigned to the GE Co., for an
invention of “ a bacterium from the genus Pseudomonas containing atleast two [four
were there] stable energy-generating plasmids, each of said plasmids providing a
separate hydrocarbon degradative pathway”.

ISSUE:
Whether respondent's microorganism constitutes a "manufacture" or
"composition of matter" within the meaning of the statute Section 101?
[Section 101:Whoever invents or discovers any new and useful process,
machine, manufacture, or composition of matter, or any new and useful
improvement thereof, may obtain a patent therefor, subject to the conditions
and requirements of this title."]
SUPREME COURT; HELD

The Patentee has produced a new bacterium with ‘markedly different’ characteristics
from any found in nature and one having the potential for significant utility. His
discovery is not nature’s handiwork, but his own.

PROPOSITIONS:

 While laws of nature, physical phenomena, and abstract ideas are not patentable,
respondent's claim is not to a hitherto unknown natural phenomenon, but to a
nonnaturally occurring manufacture or composition of matter--a product of
human ingenuity "having a distinctive name, character [and] use.
 The passage of the 1930 Plant Patent Act, which afforded patent protection to
certain asexually reproduced plants, and the 1970 Plant Variety Protection Act,
which authorized protection for certain sexually reproduced plants but excluded
bacteria from its protection, does not evidence congressional understanding that
the terms "manufacture" or "composition of matter” in section 101 do not include
living things.
 That genetic technology was unforeseen when Congress enacted section 101
require the conclusion that micro-organisms cannot qualify as patentable subject
matter until Congress expressly authorizes such protection

DISSENT:
[Because Congress thought it had to legislate in order to make agricultural
"human-made inventions" [in the light of Plant patent Act 1930 and Plant varieties
protection Act 1970] patentable and because the legislation Congress enacted is
limited, it follows that Congress never meant to make items outside the scope
of the legislation patentable. [Mr. Justice BRENNAN: But as I have shown, the

127
Court's decision does not follow the unavoidable implications of the statute. Rather,
it extends the patent system to cover living material even though Congress plainly has
legislated in the belief that section 101 does not encompass living organisms. It is
the role of Congress, not this Court, to broaden or narrow the reach of the patent
laws. This is especially true where, as here, the composition sought to be patented
uniquely implicates matters of public concern.

2. Gottschalk v. Benson, 409 U.S. 63 (1972, SC)

FACTS:

The case involved a patent application for a method of converting binary-coded


decimal (BCD) numerals into pure binary numerals to be put to use in a general
purpose digital computer. The claims were not limited to any particular art or
technology, to any particular apparatus or machinery, or to any particular end use.
They purported to cover any use of the claimed method in a general-purpose digital
computer of any type.

ISSUE:

The question was whether the method described and claimed is a 'process' within the
meaning of the Patent Act [S. 101].

SUPREME COURT:

The Supreme Court speaking through DOUGLAS J, denying the patent, held that :

“It is conceded that one may not patent an idea. But in practical effect that
would be the result if the formula for converting BCD numerals to pure
binary numerals were patented in this case. The mathematical formula
involved here has no substantial practical application except in connection
with a digital computer, which means that if the judgment below is affirmed,
the patent would wholly pre-empt the mathematical formula and in practical
effect would be a patent on the algorithm itself.”

PROPOSITIONS:

 A principle, in the abstract, is fundamental truth, an original cause, a motive,


and these cannot be patented, as no one can claim in any of them an exclusive
right.
 Phenomena of nature, although just discovered, mental processes, and abstract
intellectual concepts are not patentable as they are basic tools of scientific and
technological work.
 Here the 'process' claim [of converting BCD into pure binary numbers] is so
abstract and sweeping as to cover both known and unknown uses of the BCD
to pure binary conversion.

128
 It is argued that a process patent must either be tied to a particular machine or
apparatus or must operate to change articles or materials to a 'different state or
thing.' We do not hold that no process patent could ever qualify if it did not
meet the requirements of our prior precedents.
 It is said that the decision precludes a patent for any program servicing a
computer. We do not so hold.
 It is said that we have before us a program for a digital computer but extend
our holding to programs for analog computers. We have, however, made clear
from the start that we deal with a program only for digital computers.
 It is said we freeze process patents to old technologies, leaving no room for the
revelations of the new, onrushing technology. Such is not our purpose.
 If programs for digital computers are to be patentable, problems are raised
which only congressional committees can manage, and question is policy
matter to which court is not competent to speak.

3. Parker v. Flook, 437 U.S. 584 (1978, SC)

FACTS:
Flook applied for a patent on a method for updating alarm limits‖ in a process
comprising the catalytic chemical conversion of hydrocarbons. The process
involved three steps: an initial step which merely measures the present value
of the process variable (e. g., the temperature); an intermediate step which
uses an algorithm to calculate an updated alarm-limit value; and a final step
in which the actual alarm limit is adjusted to the updated value.
The only difference between the conventional methods of changing alarm
limits and that described in respondent's application rests in the second step--
the mathematical algorithm or formula.
ISSUE:
The patentability of the process was challenged—whether an abstract idea?
ARGUMENTS;
Flook tried to distinguish the holding in Gotschalk v. Benson in the following
way -
Respondent correctly points out that this language does not apply to his
claims. He does not seek to "wholly preempt the mathematical formula,"
since there are uses of his formula outside the petrochemical and oil-refining
industries that remain in the public domain. And he argues that the presence
of specific "post-solution" activity--the adjustment of the alarm limit to the
figure computed according to the formula--distinguishes this case from
Benson and makes his process patentable.
SUPREME COURT:
The majority speaking through STEVENS J did not agree and held –
“The notion that post-solution activity, no matter how conventional or
obvious in itself, can transform an unpatentable principle into a patentable

129
process exalts form over substance. A competent draftsman could attach
some form of post-solution activity to almost any mathematical formula; the
Pythagorean theorem would not have been patentable, or partially
patentable, because a patent application contained a final step indicating
that the formula, when solved, could be usefully applied to existing surveying
techniques. The concept of patentable subject matter under s. 101 is not "like
a nose of wax which may be turned and twisted in any direction.”
Holding that such a process is un-patentable, they said –
Respondent's process is unpatentable under S 101, not because it contains a
mathematical algorithm as one component, but because once that algorithm
is assumed to be within the prior art, the application, considered as a whole,
contains no patentable invention. Even though a phenomenon of nature or
mathematical formula may be well known, an inventive application of the
principle may be patented. Conversely, the discovery of such a phenomenon
cannot support a patent unless there is some other inventive concept in its
application.
‘Respondent's method for updating alarm limits during catalytic conversion
processes, in which the only novel feature is a mathematical formula, and is
not patentable under S 101 of the Patent Act. The chemical processes
involved in catalytic conversion are well known, as are the monitoring of
process variables, the use of alarm limits to trigger alarms, the notion that
alarm limit values must be recomputed and readjusted, and the use of
computers for automatic process monitoring’.
PROPOSITIONS:
 Process is not unpatentable simply because it contains law of nature or
mathematical algorithm; however, process itself, not merely
mathematical algorithm, must be new and useful.
 The line between a patentable "process" and an unpatentable "principle"
is not always clear. Both are "conception[s] of the mind, seen only by
[their] effects when being executed or performed."
 Identification of postsolution applications of mathematical formula did
not make method eligible for patent protection.
 The holding [in Gotschalk v. Benson] that the discovery of that method
[conversion of BSD into pure binary numbers] could not be patented as
a process forecloses a purely literal reading of S 101. Reasoning that an
algorithm, or mathematical formula, is like a law of nature, Benson
applied the established rule that a law of nature cannot be the subject of
a patent.
 Novelty of mathematical formula or algorithm is not determining factor
as to patentability of process employing algorithm; whether algorithm
was in fact known or unknown at time of claimed invention as one of
basic tools of scientific and technological works, it is treated as though it
were familiar part of prior art.

130
 Rule that discovery of law of nature cannot be patented rests, not on the
notion that natural phenomena are not processes, but rather on the more
fundamental understanding that they are not kind of discoveries‖ that
patent law was enacted to protect. The mere recognition‖ of theretofore
existing phenomenon or relationship carries with it no rights to exclude
others from its enjoyment.

4. Diamond v. Diehr, 450 U.S. 175 (1981, SC).

FACTS:

Respondents filed a patent application claiming invention for a process for


molding raw, uncured synthetic rubber into cured precision products. While it was
possible, by using well-known time, temperature and cure relationships, to
calculate by means of an established mathematical equation (Arhenius Equation)
when to open the molding press and remove the cured product, according to
respondents the industry had not been able to measure precisely the temperature
inside the press, thus making it difficult to make the necessary computations to
determine the proper cure time. Respondents characterized their contribution to
the art to reside in the process of constantly measuring the temperature inside the
mold and feeding the temperature measurements into a computer that repeatedly
recalculates the cure time by use of the Arhenius equation and then signals a
device to open the press at the proper time. This solved the problem of under-
curing and over-curing which was a persistent problem in the industry.

ISSUE:

The issue was whether the claimed process of the Respondent fell within the four
corners of patentable subject matter under S. 101.

SUPREME COURT:

Differentiating the earlier cases, and upholding the patentability of the claim, the
Court speaking through REHNQUIST J held –

“In contrast, the respondents here do not seek to patent a mathematical formula.
Instead, they seek patent protection for a process of curing synthetic rubber. Their
process admittedly employs a well-known mathematical equation, but they do not
seek to pre-empt the use of that equation. Rather, they seek only to foreclose from
others the use of that equation in conjunction with all of the other steps in their
claimed process. These include installing rubber in a press, closing the mold,
constantly determining the temperature of the mold, constantly recalculating the
appropriate cure time through the use of the formula and a digital computer, and
automatically opening the press at the proper time. Obviously, one does not need
a "computer" to cure natural or synthetic rubber, but if the computer use
incorporated in the process patent significantly lessens the possibility of

131
"overcuring" or "undercuring," the process as a whole does not thereby become
unpatentable subject matter.”

Specifically on Parker v. Flook, the Court impliedly over-ruled it‘s earlier


holding, though explicitly it only tried to limit its ruling and differentiate it –

“In determining the eligibility of respondents' claimed process for patent


protection under S 101, their claims must be considered as a whole. It is inappropriate
to dissect the claims into old and new elements and then to ignore the presence of the
old elements in the analysis. This is particularly true in a process claim because a new
combination of steps in a process may be patentable even though all the constituents
of the combination were well known and in common use before the combination was
made. The "novelty" of any element or steps in a process, or even of the process
itself, is of no relevance in determining whether the subject matter of a claim falls
within the S 101 categories of possibly patentable subject matter.”

“It is argued that the procedure of dissecting a claim into old and new elements is
mandated by our decision in Flook which noted that a mathematical algorithm must
be assumed to be within the "prior art." It is from this language that the petitioner
premises his argument that if everything other than the algorithm is determined to be
old in the art, then the claim cannot recite statutory subject matter. The fallacy in this
argument is that we did not hold in Flook that the mathematical algorithm could not
be considered at all when making the S 101 determination. To accept the analysis
proffered by the petitioner would, if carried to its extreme, make all inventions
unpatentable because all inventions can be reduced to underlying principles of nature
which, once known, make their implementation obvious. The analysis suggested by
the petitioner would also undermine our earlier decisions regarding the criteria to
consider in determining the eligibility of a process for patent protection.”

PROPOSITIONS LAID DOWN –

 In enacting the Patent Act Congress intended the statutory subject matter to
include anything under the sun that is made by man.
 For purpose of Patent Act, a "process" is an act or series of acts performed on
the subject matter to be transformed and reduced to a different state or thing,
and if new and useful it is just as patentable as a piece of machinery
regardless of whether the machinery pointed out as suitable to perform the
process is new or patentable. This is the transformative test for determining
patentable subject matter.
 A claim drawn to subject matter otherwise statutory does not become
nonstatutory, i. e., outside Patent Act's categories of possibly patentable
subject matter, simply because it uses a mathematical formula, computer
program or digital computer.
 An application of a law of nature or mathematical formula to a known
structure or process may well be deserving of patent protection.
 Although by itself, the Arrhenius equation to calculate cure time in rubber
molding presses is not patentable, when a process for curing rubber is devised
which incorporates it in a more efficient solution of the equation, such

132
process is at the very least not barred at the threshold as outside Patent Act's
categories of possibly patentable subject matter.
 In determining eligibility of claimed process for curing synthetic rubber
including in several of its steps use of a mathematical formula and a
programmed digital computer, the claims were to be considered as a whole
and, it was inappropriate to dissect the claims into old and new elements and
then ignore the presence of the old elements in the analysis.
 The "novelty" of any element or steps in a process, or even of the claimed
process itself, is of no relevance in determining whether the subject matter of
a claim falls within Patent Act's categories of possibly patentable subject
matter.
 Whether a particular invention is novel is a question fully apart from whether
the invention falls into a statutory category of possibly patentable subject
matter.

5. In Re Allappat, 33 F.3d 1526, (1994, CA, Federal Circuit).


Alappat's invention related generally to use of a machine (rastorizer) as a
means for creating a smooth waveform display in a digital oscilloscope. The
claim in issue here was claim 15, that read –
“A rasterizer for converting vector list data representing sample magnitudes
of an input waveform into anti-aliased pixel illumination intensity data to be
displayed on a display means comprising:
(a) means for determining the vertical distance between the endpoints of
each of the vectors in the data list;
(b) means for determining the elevation of a row of pixels that is spanned by
the vector;
(c) means for normalizing the vertical distance and elevation; and
(d) means for outputting illumination intensity data as a predetermined
function of the normalized vertical distance and elevation.‖
The Court held that the invention falls within the patentable subject matter under S
101 and relying upon Diamond v. Diehr said –

“[The Court in Diamond v. Diehr] never intended to create an overly broad,


fourth category of [mathematical] subject matter excluded from S 101. Rather,
at the core of the Court's analysis ... lies an attempt by the Court to explain a
rather straightforward concept, namely, that certain types of mathematical
subject matter, standing alone, represent nothing more than abstract ideas until
reduced to some type of practical application, and thus that subject matter is
not, in and of itself, entitled to patent protection.”

“The proper inquiry in dealing with the so called mathematical subject matter
exception to S 101 alleged herein is to see whether the claimed subject matter
as a whole is a disembodied mathematical concept, whether categorized as a
mathematical formula, mathematical equation, mathematical algorithm, or the
like, which in essence represents nothing more than a "law of nature," "natural

133
phenomenon," or "abstract idea." If so, Diehr precludes the patenting of that
subject matter. That is not the case here. Although many, or arguably even all,
of the means elements recited in claim 15 represent circuitry elements that
perform mathematical calculations, which is essentially true of all digital
electrical circuits, the claimed invention as a whole is directed to a combination
of interrelated elements which combine to form a machine for converting
discrete waveform data samples into anti-aliased pixel illumination intensity
data to be displayed on a display means. This is not a disembodied
mathematical concept which may be characterized as an "abstract idea," but
rather a specific machine to produce a useful, concrete, and tangible result.”

6. State Street Bank v. Signature Financial Group, 149 F.3d 1368


State Street Bank brought an action against assignee of patent for
computerized accounting system used to manage mutual fund investment
structure, seeking declaratory judgment that patent was invalid and
unenforceable. The patent granted was directed to a data processing system (the
system) for implementing an investment structure which was developed for use
in Signature's business as an administrator and accounting agent for mutual
funds. In essence, the system, identified by the proprietary name Hub and
Spoke®, facilitates a structure whereby mutual funds (Spokes) pool their assets
in an investment portfolio (Hub) organized as a partnership. This investment
configuration provides the administrator of a mutual fund with the advantageous
combination of economies of scale in administering investments coupled with
the tax advantages of a partnership.
The United States District Court for the District of Massachusetts, granted
summary judgment for bank, and assignee appealed. The Court of Appeals,
RICH J., held that:
1. The invention was not unpatentable under mathematical algorithm
exception to patentability – “Transformation of data, representing discrete dollar
amounts, by a machine through a series of mathematical calculations into a final
share price, for purpose of managing mutual fund investment structure, was
practical application of a mathematical algorithm, formula, or calculation,
because it produced useful, concrete and tangible result, and claimed machine
thus was not unpatentable under mathematical algorithm exception to
patentability.‖ ―Unpatentable mathematical algorithms are identifiable by
showing they are merely abstract ideas constituting disembodied concepts or
truths that are not 'useful.' ... [T]o be patentable an algorithm must be applied in
a 'useful' way.‖
2. Freeman-Walter-Abele Test, developed by the CCPA is wrong
“The FWA test (developed by the CCPA in 1978, 1980) has been thus
articulated: First, the claim is analyzed to determine whether a mathematical
algorithm is directly or indirectly recited. Next, if a mathematical algorithm is
found, the claim as a whole is further analyzed to determine whether the

134
algorithm is "applied in any manner to physical elements or process steps," and,
if it is, it ‗passes muster under S 101.”
“After Diehr and Chakrabarty, the Freeman-Walter-Abele test has little, if
any, applicability to determining the presence of statutory subject matter.”
3. There need not be a ―physical‖ result for the invention to be patentable
“A machine programmed with the Hub and Spoke software and admittedly
produces a useful, concrete, and tangible result…this renders it statutory subject
matter, even if the useful result is expressed in numbers, such as price, profit,
percentage, cost, or loss.”
4. There is no "business method" exception to patentability.
“[Patentability does not turn on whether the claimed method does "business"
instead of something else, but on whether the method, viewed as a whole, meets
the requirements of patentability as set forth in Sections 102, 103, and 112 of
the Patent Act]…We take this opportunity to lay this ill-conceived exception to
rest.”

7. AT & T Corp. v. Excel Communications, 172 F.3d 1352


AT&T held patent relating to “process for indicating telephone call
recipient's primary interexchange carrier (PIC) as data field in standard message
record, which employed subscribers' and call recipients' PICs as data, and
applied Boolean algebra to those data to determine value of PIC indicator, and
applied that value through switching and recording mechanisms to create
signals. The addition of the indicator aids long-distance carriers in providing
differential billing treatment for subscribers, depending upon whether a
subscriber calls someone with the same or a different long-distance carrier.”
This patent was challenged by a competitior [Excel Communications] in
response to an infringement action brought by AT & T. Deciding on the
patentability of the process, it was held that this was within scope of patentable
subject matter, as this process applied Boolean principle to produce useful,
concrete, tangible result without preempting other uses of the mathematical
principle. Further, the fact that there was no ―physical transformation‖ in the
process was held irrelevant for an enquiry into patentability.
PROPOSITIONS LAID DOWN
 “Any step-by-step process, be it electronic, chemical, or mechanical,
involves an "algorithm" in the broad sense of the term.”
 “In his dissent in Diehr, Justice Stevens noted two concerns regarding
the S 101 issue, and to which, in his view, federal judges have a duty to
respond… His second concern, that the ambiguous concept of
"algorithm" could be used to make any process unpatentable, can be laid
to rest once the focus is understood to be not on whether there is a
mathematical algorithm at work, but on whether the algorithm-
containing invention, as a whole, produces a tangible, useful, result.”

135
 “Patent claims containing algorithms need not involve physical
transformation or conversion of subject matter from one state into
another to be deemed patentable subject matter.”
 “The notion of "physical transformation" can be misunderstood. In the
first place, it is not an invariable requirement, but merely one example
of how a mathematical algorithm may bring about a useful application.”
 “[T]his type of physical limitations analysis seems of little value
because after Diehr and Alappat, the mere fact that a claimed invention
involves inputting numbers, calculating numbers, outputting numbers,
and storing numbers, in and of itself, would not render it nonstatutory
subject matter, unless, of course, its operation does not produce a
'useful, concrete and tangible result‘.”
 “Whether stated implicitly or explicitly, we consider the scope of S 101
to be the same regardless of the form--machine or process--in which a
particular claim is drafted.”

8. Bernard L. Bilski and Rand A. Warsaw, v. David J. Kappos, Under


Secretary of Commerce for Intellectual Property and Director, Patent and
Trademark Office.(On writ of Certiorari to the United States Court of Appeals
for the Federal Circuit)Judgment delivered – June 28th, 2010
Facts – 1 Bilski‘s application broadly claimed a series of steps for hedging risks
involved in commodity transactions. It made a specific claim for the use of these series
of steps with respect to transactions where ‘energy’ is the commodity.

2. The application was rejected by the examiner on grounds that it was not directed
to ‘technological arts’ since (1) the steps had not been implemented on a specific
apparatus and (2) it merely manipulates an abstract idea without limiting the use of the
series of steps to a particular application.

3. The examiner‘s decision was affirmed by the Board of Patent Appeals and
Interferences (BPAI) but his contention that the invention must relate to technological
arts or must be performed on a machine to be eligible for a patent was rejected. The
BPAI held the application disclosed a mere abstract idea which did not transform
physical subject matter.

The Federal Circuit (FC) held the application to be ineligible on grounds that it was
neither tied to a machine nor did it transform an article into a particular state or a thing,
that is that machine-transformation test. It was concluded that the machine-or-
transformation test is the sole test to determine the patent eligibility, and since this
application did not satisfy the test, it was rejected.

US Supreme Court :

136
Justice Kennedy: The opinion is concerned with three aspects of the case- 1)
exclusive application or otherwise of the MT test, 2) patent eligibility or otherwise of a
patent application which claims a business method and 3) that the application merely
discloses an abstract idea.

MT Test

Firstly, Justice Kennedy speaking for the court states that this test should not be
construed as the sole/exclusive test for determining the patent eligibility of an
application since such exclusive application of the test would preclude genuinely patent
eligible efforts from the scope of section 101 of the US Patents Act (which permits
grant of a patent for processes, machines, manufactures, composition of matter, or
material). A dynamic interpretation of Section 101 is necessary to ensure that
―unforeseen innovations‖ are covered by section 101. However, it is clarified that the
Court in this judgment does not attempt to define parameters for a new test.

Business Method Application

The second issue of status of business method applications. It is contended


that since process has been expressly defined in the Act, such an express
definition provided in the Act should prevail over other interpretations.
Therefore, the contention that the term ―process‖ must be given a restrictive
interpretation since the neighbouring terms used in section 101 are machine,
manufacture, composition of matter, material is rejected.
According to Justice Kennedy, business methods receive support from
Section 273(b) of the Act. Section 273(b) provides an alleged an infringer a
defense against infringement on grounds that the ―method in the patent‖ had
been used prior to the filing of the patent. Section 273(a)(3) also defines method
for the purpose of this defense, as a method for doing or conducting business.
Thus, it is averred that business methods are patent eligible, but the standards
used for evaluating these patents should be higher considering the nature of
subject matter dealt with by such applications.
Abstract Idea
It is concluded that the Bilski application is an abstract idea, and thus tests
laid down in the Gottschalk v. Benson, Parker v. Flook and Diamond v. Diehr
would be sufficient for the purpose of this application, without the need for the
Supreme Court to come out with a different test.
Justice Kennedy compares the broad claims on hedging to the claims in
Benson and on this basis holds that the application is a mere abstract idea. Thus,
Justice Kennedy has relied on the Benson and Flook cases together, to arrive at
the conclusion.
Justice Steven‘s Opinion
Justice Stevens opines that merely because a series of steps are specific and
not abstract the steps do not constitute a process under section 101. He further

137
contends that business methods are ineligible subject-matter for patents, and the
same is sought to be proved through use of historical and statutory approaches.
Section 273 was included in the Act at a time when the Federal Circuit Court
was beginning to consider allowing business method patents. To ensure that the
long history behind non-grant of patents to business methods is not overturned
by this sudden change of course by the Federal Circuit, section 273 was
included i.e. to reinforce the commonly-held view among practitioners and
applicants that business methods were hitherto unpatentable and remain
unpatentable; consequently, under section 273 their ineligibility is available as a
defense against an allegation of infringement by a patentee whose patent claims
a business method.
Therefore, he says section 273 must be used to argue against business
method patents.
He further argues that section 101 requires the subject-matter of a patent
application to be new and useful, applying literal interpretation it means that
applications which are useful for conducting one‘s business are eligible for a
patent grant, but applications which deal with the very method of doing or
conducting his business are not eligible.
According to Justice Kennedy, imputing an ordinary, contemporary and
common meaning to the word ‘process’ as used in the statute is a fine principle
of statutory interpretation, however it should give way to a specific connotation
that ―process‖ has come to acquire by virtue of its use in an extremely
technical and specialized setting and which is the product of more than a
century‘s jurisprudence.
To understand the term process, Justice Stevens refers to Justice Dyk‘s
review (in the Federal Circuit Bilski decision) of the evolution of American and
English precedents on patents. In the Federal Circuit Decision, Justice Dyk
concludes that at no point do any of the precedents suggest eligibility of
business methods; this, despite the fact that innovation in ways of doing or
conducting business is not a new phenomenon.
In fact, almost a century ago, the changes made to business models were
revolutionary. Justice Stevens uses this historical fact to establish that despite
there being several innovations by entrepreneurs in the manner in which they
conducted their business, barring one-off instances which were never tested
for validity because they never reached the Court, business methods were
not even contemplated or deliberated as being within the scope of patentable
subject-matter.
He proposes that instead of approving the eligibility of business method
application for grant of patent and rejecting the Bilski application as invalid for
claiming an abstract idea, the court should have held that since the Bilski
application claims a business method, it is unpatentable, regardless of it being
abstract.
Justice Breyer‘s Opinion

138
Justice Breyer concurs with Justice Stevens that the Bilski application must
be rejected on grounds of claiming a business method since a since method is
not a patentable process within the meaning of ―process‖ under Section 101.
He expresses that the purpose of his opinion is to summarise the points of
law commonly agreed upon by all the Judges, which are as follows:
1. The Bilski application undeniably claims an abstract idea
2. Although section 101 is broad, it is not without limitations. The judicially-
developed limitations are phenomena of nature, mental processes and abstract
ideas. These limitations are imperative because the purpose of patent law is not
restricted to determining what merits a patent, but also what is free for all to use.
3. He concedes that the MT test has helped the Court is several occasions in
the past and cites Diehr and Benson to make his point.
4. That said, the MT test has always been an important clue and was never
intended to be the sole test of patent eligibility.
5. He again qualifies the above observation that just because the MT test is
not be treated as the sole test, it does not follow that anything which produces a
useful, concrete and tangible result is eligible for a patent grant.
6. He concludes finally that the opinion of the Court is not meant to
undervalue the utility of the MT test nor to state that certain technologies lie
beyond its reach.

9. VICOM‘S System Inc‘s Application [(T-208/84); [1987] 2 EPOR 74]


In this case the Invention was a ‘method and apparatus for improving the
quality of pictures and speeding up their processing’ The board noted that a
mathematical algorithm as such is a abstract concept and does not have any
direct technical result. Whereas a technical process results in a change to a
physical entity and this process may be carried out by a computer having
suitable hardware or a general purpose computer that is programmed.
So a claim directed to technical process which is carried out under the
control of a program (whether the implementation is in hardware or in
software) should be allowable under Article 52(2)(c) and (3) EPC since it
cannot be regarded as relating to a computer program as such.
PROPOSITIONS LAID DOWN
The board went on to hold that merely storing a program in a computer
does not render the combination patentable. Rather, "something further is
necessary .... the decisive inquiry is what technical contribution the invention
makes to the known art.' There must, ... be some technical advance on the
prior art in the form of a new result e.g., a substantial increase in processing
speed as in Vicom.

10. IBM /TEXT PROCESSING [(1999) EPOR 301]


In this case the application related to patenting of a data processing system
used to display information in windows in such a manner that the information in

139
one window moved automatically to new position, when obstructed by another
window.
PROPOSITIONS LAID DOWN
1. ‘As such’ = programs that did not produce ―technical effect‖
2. Technical effect=not merely that they are used to operate computers, but
found in ‗further effects‘
3. Programs had ‘potential to produce technical effect ―it is illogical to
grant patent both for Method and apparatus adopted for carrying out the method
but NOT for
computer program product comprising all features enabling implementation
of method…”

11. FUJITSU‘S APPLICATION, [{1997} RPC 608 (CA)]


In this case Justice Laddie summarized the position on the issue of software
patents
PROPOSITIONS LAID DOWN
Held that it is not permissible to seek protection for a computer program
when stored in a magnetic medium or when merely loaded into a computer.
Prima-facie a computer running under the control of one program is different
piece of apparatus from the same computer when running under the control of
another program. A claim to computer when controlled by a program or to a
method of controlling the computer by a program or to a method of carrying out
the process by use of a computer so controlled can be subject matter of patent
protection. But the court and the patent office has to direct its intention not to
the fact that the program is controlling the computer, but to what the computer
so controlled is doing.

12. ALICE CORPORATION V. CLS BANK INTERNATIONAL


Facts: Alice owns four patents which relate to a computerized trading
platform used for conducting financial transactions in which a third party
settles obligations between a first and a second party so as to eliminate
"counterparty" or "settlement" risk. The claim pertains to:
1. computerized methods,
2. computer-readable media, and
3. systems that are useful for conducting these financial transactions.
CLS filed suit against Alice seeking a declaratory judgment of non-
infringement, invalidity, and unenforceability of these patents. The district
court adjudicated in favour of CLS by concluding that Alice's method claims
are directed to an abstract idea of employing an intermediary to facilitate
simultaneous exchange of obligations in order to minimize risk. On July 9,
2012, a panel of CAFC reversed, holding that the claims at issue, including
claims drawn to methods, computer-readable media, and systems, were all
patent eligible under S.101. CLS filed a petition for rehearing en banc, which

140
was granted on October 9, 2012. Upon consideration en banc, a majority of the
court affirmed the district court’s holding that the asserted method and
computer-readable media claims are not directed to eligible subject matter
under 35 U.S.C. § 101. An equally divided court affirmed the district court’s
holding that the asserted system claims are not directed to eligible subject
matter under that statute.
As per LOURIE, Circuit Judge, concurring, with whom Circuit Judges
DYK, PROST, REYNA, and WALLACH join.
Method Claims:
The question for consideration was whether that process amounts to no
more than a patent-ineligible abstract idea? The claim related to a method of
reducing settlement risk by effecting trades through a third-party intermediary
(here, the supervisory institution) empowered to verify that both parties can
fulfill their obligations before allowing the exchange—i.e., a form of escrow.
Apart from the idea of third-party intermediation, the claim’s substantive
limitations require creating shadow records, using a computer to adjust and
maintain those shadow records, and reconciling shadow records and
corresponding exchange institution accounts through end-of-day transactions.
Held that “In sum, there is nothing in the asserted method claims that
represents “significantly more” than the underlying abstract idea for
purposes of § 101. But for the implied requirement for computer
implementation, the broad, non-technical method claims presented here
closely resemble those in Bilski, which also explained a “basic concept of . . .
protecting against risk.” And, as described, adding generic computer
functions to facilitate performance provides no substantial limitation and
therefore is not “enough” to satisfy § 101. As in Bilski, upholding Alice’s
claims to methods of financial intermediation “would pre-empt use of this
approach in all fields, and would effectively grant a monopoly overan abstract
idea.” Consequently, the method claims are drawn to patent-ineligible subject
matter and invalid under § 101.”
Computer-Readable Medium Claims
It was held with respect to computer readable medium claims that they
were merely method claims in the guise of a device and thus do not overcome
the Supreme Court’s warning to avoid permitting a “competent draftsman” to
endow abstract claims with patent-eligible status.
System Claims
The claims related to “data processing systems” configured to carry out a
series of steps that mirror Alice’s method claims—maintaining shadow
records, allowing only those transactions supported by adequate value in the
shadow records, adjusting the shadow records pursuant to such transactions,
and later instructing exchange institutions to execute the allowed transactions.
Machines are unquestionably eligible for patenting but in this case the claim
was not directed for a computer per se but towards abstract methods coupled
with computers adapted to perform those methods. Held that, abstract methods

141
do not become patent-eligible machines by being clothed in computer
language.
Present Status: After the en banc decision by US CAFC, the US Supreme
Court has granted certiorari and briefs have already been filed in SC and the
case is listed for arguments on 31/03/2014 to decide the issue whether claims
to computer-implemented inventions – including claims to systems and
machines, processes, and items of manufacture, are directed to patent-eligible
subject matter within the meaning of 35 U.S.C. § 101?

B. Natural laws- Unpatentable subject matter


An application of natural law to become patent eligible must involve
something than merely saying “apply it”

13. Mayo Collaborative Services v. Prometheus Laboratories, Inc.


Decided March 20, 2012 by US SC
Facts: Respondent, Prometheus, is the sole and exclusive licensee of the two
patents at issue, which concern the use of thiopurine drugs to treat autoimmune
diseases. When ingested, the body metabolizes the drugs, producing metabolites
in the bloodstream. Because patients metabolize these drugs differently, doctors
have found it difficult to determine whether a particular patient’s dose is too
high, risking harmful side effects, or too low, and so likely ineffective. The
patent claims in issue set forth processes embodying researchers’ findings that
identify correlations between metabolite levels and likely harm or
ineffectiveness with precision. Each claim recites (1) an “administering” step—
instructing a doctor to administer the drug to his patient—(2) a “determining”
step—telling the doctor to measure the resulting metabolite levels in the
patient’s blood—and (3) a “wherein” step—describing the metabolite concen-
trations above which there is a likelihood of harmful side-effects and below
which it is likely that the drug dosage is ineffective, and informing the doctor
that metabolite concentrations above or below these thresholds “indicate a
need” to decrease or increase (respectively) the drug dosage. Mayo bought and
used diagnostic tests based on Prometheus’ patents. But in 2004 Mayo
announced that it intended to sell and market its own, somewhat different,
diagnostic test. Prometheus sued Mayo contending that Mayo’s test infringed its
patents. The District Court found that the test infringed the patents but granted
summary judgment to Mayo, reasoning that the processes claimed by the pa-
tents effectively claim natural laws or natural phenomena—namely, the
correlations between thiopurine metabolite levels and the toxicity and efficacy
of thiopurine drugs—and therefore are not patentable. The Federal Circuit
reversed, finding the processes to be patent eligible under the Circuit’s
“machine or transformation test.” So Mayo filed a writ of certiorari which was
granted for reconsideration in light of Bilski v. Kappos.
Held:
a. Prometheus’ process is not patent eligible.

142
The established position is that laws of nature, natural phenomena, and
abstract ideas are not patentable and to transform an unpatentable law of nature
into a patent-eligible application of such a law, one must do more than simply
state the law of nature while adding the words “apply it.”
Prometheus’ patents set forth laws of nature—namely, relationships between
concentrations of certain metabolites in the blood and the likelihood that a
dosage of a thiopurine drug will prove ineffective or cause harm. While it takes
a human action (the administration of a thiopurine drug) to trigger a
manifestation of this relation in a particular person, the relation itself exists in
principle apart from any human action. The relation is a consequence of the
ways in which thiopurine compounds are metabolized by the body—entirely
natural processes. And so a patent that simply describes that relation sets forth a
natural law.
The administering step, determining step, and wherein step are not
themselves natural laws but neither are they sufficient to transform the nature of
the claim. Beyond picking out the relevant audience, namely those who
administer doses of thiopurine drugs, the claim simply tells doctors to: (1)
measure (somehow) the current level of the relevant metabolite, (2) use
particular (unpatentable) laws of nature to calculate the current
toxicity/inefficacy limits, and (3) reconsider the drug dosage in light of the law.
These instructions add nothing specific to the laws of nature other than what is
well-understood, routine, conventional activity, previously engaged in by those
in the field. And since they are steps that must be taken in order to apply the
laws in question, the effect is simply to tell doctors to apply the law somehow
when treating their patients.
b. If a law of nature is not patentable, then neither is a process reciting a
law of nature, unless that process has additional features that provide practical
assurance that the process is more than a drafting effort designed to
monopolize the law of nature itself.

14. Association for Molecular Pathology v. Myriad Genetics, Inc.


US SC, Decided on June 13, 2013
Facts: Myriad Genetics, Inc. obtained several patents after discovering the
precise location and sequence of the BRCA1 and BRCA2 genes, mutations of
which can dramatically increase the risk of breast and ovarian cancer. This
knowledge allowed Myriad to determine the genes’ typical nucleotide sequence,
which, in turn, enabled it to develop medical tests useful for detecting mutations
in these genes in a particular patient to assess the patient’s cancer risk.
Petitioners filed suit, seeking a declaration that Myriad’s patents are invalid
under 35 U. S. C. §101. As relevant here, the District Court granted summary
judgment to petitioners, concluding that Myriad’s claims were invalid because
they covered products of nature. The Federal Circuit initially reversed but on
remand in light of Mayo Collaborative Services v. Prometheus Laboratories,
Inc., the Circuit found both isolated DNA and cDNA patent eligible.

143
Held:
1. A naturally occurring DNA segment is a product of nature and not patent
eligible merely because it has been isolated, but cDNA is patent eligible
because it is not naturally occurring.
2. Myriad’s DNA claim falls within the law of nature exception. Myriad’s
principal contribution was uncovering the precise location and genetic
sequence of the BRCA1 and BRCA2 genes.
Myriad did not create or alter either the genetic information encoded in the
BCRA1 and BCRA2 genes or the genetic structure of the DNA. It found an im-
portant and useful gene, but ground breaking, innovative, or even brilliant
discovery does not by itself satisfy the S. 101 inquiry. Finding the location of
the BRCA1 and BRCA2 genes does not render the genes patent eligible
“new . . . composition[s] of matter,”
3. cDNA is not a “product of nature,” so it is patent eligible under S. 101.
cDNA does not present the same obstacles to patentability as naturally
occurring, isolated DNA segments. Its creation results in an exons-only
molecule, which is not naturally occurring. Its order of the exons may be
dictated by nature, but the lab technician unquestionably creates something
new when introns are removed from a DNA sequence to make cDNA.

C. NOVELTY
15. Continental Can Co. USA v. Monsanto Co. 948 F.2d 1264 (Fed. Cir. 1991)
This case lays down the proper test for determining whether a prior art
disclosure inherently anticipates a patent. This test is popularly called as the test
of ―inherent anticipation.
Continental Can Co. (―Continental‖) had patented a hollow ribbed bottom
for plastic bottles [Conobase ‗324]. Monsanto argued that Continental's patent
was invalid as anticipated by a previous patent on a bottom for plastic bottles
(the Marcus patent). The Marcus patent however was only for solid bottom, and
did not explicitly talk of hollow ribbed bottom. Monsanto argued that
anticipation lies because the Marcus patent's ribs are "inherently" hollow,
regardless of how they are shown in the Marcus patent. This is because the
Marcus ribs are formed by injection blow molding, which is the same process
described for the Conobase '324 ribs, and therefore hollow ribs are inherently
disclosed in the Marcus patent.
The court, recognizing the heavy factual nature of the arguments, remanded
the case back for a factual determination.
Explaining the doctrine of ―inherency‖, as a test for ―Novelty‖ it said –
―To serve as anticipation when the reference is silent about the asserted
inherent characteristic, such gap in the reference may be filled with recourse to
extrinsic evidence. Such evidence must make clear that the missing descriptive
matter is necessarily present in the thing described in the reference, and that it
would be so recognized by persons of ordinary skill. [Citations omitted.]

144
Inherency, however, may not be established by probabilities or possibilities.
The mere fact that a certain thing may result from a given set of circumstances
is not sufficient. If, however, the disclosure is sufficient to show that the natural
result flowing from the operation as taught would result in the performance of
the questioned function, it seems to be well settled that the disclosure should be
regarded as sufficient.
This modest flexibility in the rule that "anticipation" requires that every
element of the claims appear in a single reference accommodates situations
where the common knowledge of technologists is not recorded in the reference;
that is, where technological facts are known to those in the field of the
invention, albeit not known to judges. It is not, however, a substitute for
determination of patentability in terms of S 103 [Non-Obviousness].”

16. General Tire & Rubber Company Limited V. Firestone Tyre & Rubber
Company Limited, [[1971] F.S.R. 417]
The patent in suit was for a process for making a compound suitable for tyre
treads by mixing synthetic rubber with oil and carbon black (a mixture
commonly referred to as "oil-extended" rubber) and for the compound thus
made. It was common ground that after the plaintiffs publicized the process in
June 1951; it was rapidly taken into general industrial use. In the present case,
the respondent plaintiffs asserted claims for the infringement of patent priority
date 20th November 1950) and the appellant defendants sought its revocation.
The invalidity of the patent was claimed on the grounds of novelty and non-
obviousness. The court dismissed the appeal and held that the prior art
publications cited by the Appellants–Defendant did not that anticipate the
present invention. Moreover, that the invention was not obvious, and that the
objection of ambiguity was not established.
As regards novelty, Sachs L.J. stated that, ―to determine whether a
patentee‘s claim had been anticipated by an earlier publication it is necessary to
compare the earlier publication with the patentee‘s claim. The earlier
publication must, for this purpose be interpreted as at the date of publication,
having regard to the surrounding circumstances which then existed, and without
regard to subsequent events. .. If the earlier publication, so construed, discloses
the same device as the device which the patentee by his claim, so construed,
asserts that he invented, the patentee‘s claim has been anticipated, but not
otherwise.
As regards, anticipation or the purposes of non-obviousness the court stated
that, ―That, in determining whether or not a patentee's claim had been
anticipated by an earlier publication, the publication and the claim had each to
be construed as at their respective relevant dates by a reader skilled in the art,
which reader could, if the art was one having a highly developed technology, be
a team. If the prior publication contained a clear description of, or clear
instructions to do or make, something that would infringe the patentee's claim if
carried out after grant of the patentee's patent, the claim would be anticipated. If

145
carrying out the directions in the prior publication would inevitably result in
something being made or done which would infringe there would be
anticipation. If, on the other hand, the prior publication contained a direction
which was capable of being carried out in a manner which would infringe, but
would be at least as likely to be carried out in a way which would not do so, the
patentee's claim would not be anticipated, although it might fail on the ground
of obviousness. To anticipate the claim, the prior publication had to contain
clear and unmistakeable direction to do what the patentee claimed to have
invented. There was on the facts of the case no anticipation by any of the cited
documents.‖ Thus, if the prior document is enabling so that by carrying out
the directions it inevitably leads to the flag being planted within the claim,
the patent will lack novelty.
Further, regarding the parameter of obviousness the court stated that, ―The
court doubted whether two of the documents relied on would have been
discovered in the course of a diligent search, but, on any view of the law, the
appellants had failed by a considerable margin to establish their plea of
obviousness. The plaintiff's commercial success was of value on that issue.”

17. Asahi Kasei Kogyo, 1991 RPC 485 (H.L.)


In this case the House of Lords held that the ―relevant‖ matter should
amount to an ―enabling disclosure‖; put shortly, sufficient information has to
be provided for the recipient to be able to put the invention into practice
himself. Applying this dicta to the facts at hand the court held that the earlier art
merely disclosed a genetic structure of the HNTF (Human Tissue Necrosis
Factor) but did not disclose the process of obtaining the same and thus was not
enabling disclosure.
As regards the capability of a person skilled in the art and the approach to be
adopted by him, the court stated that:
―Now the question is what must be the antecedent statement? I
apprehend the principle is correctly expressed: the antecedent statement
must be such that a person of ordinary knowledge of the subject would
at once perceive, understand and be able to practically apply the
discovery without the necessity of making further experiments and
gaining further information before the invention can be made useful. If
something remains to be ascertained which is necessary for useful
application of the discovery that affords sufficient room for another
valid patent.”

18. In re Leo M. HALL, [781 F.2d 897, 228 U.S.P.Q. 453, United States Court
of Appeals, Federal Circuit.]
The present case was an appeal against the decision of the USPTO, rejecting
the re issue of patent on the ground of lack of novelty. The patent was said to be
anticipated by Foldi‘s doctoral thesis. The question before the court was
whether the thesis is available as such a printed publication.

146
The facts on record showed that Foldi‘s dissertation was available at the
library o the Freiburg University, Germany. The copies of the same were freely
available to the students and faculty members of the university. As stated above
at the USPTO the patent examiner stated that on the basis of the instant record it
is reasonable to assume that the Foldi thesis was available prior to February 27,
1979. (The plaintiff relied on the dicta of In Re Bayer for his arguments)
The matter came before the Court of Appeals. The appellant argued that prior
publication bar required that the publication should be accessible to the
interested public and for this purpose it should be properly indexed and
catalogued; the Foldi thesis did not satisfy this requirement. Moreover, even if
the first requirement was satisfied, a presence of a single catalogued thesis in
one university library does not constitute sufficient accessibility of the
publication‘s teachings to those interested in the art exercising reasonable
diligence.
In determining the case the court observed that ‗public accessibility‘ has
been called the touchstone in determining whether a reference constitutes
printed publication under the novelty requirement.
Differentiating the facts in the present case, the court concluded that the
evidence at hand showed that Foldi‘s dissertation was available in the library
prior to the critical date and the normal procedures of indexing, cataloguing etc
were followed. Thus, the court concluded that there was public accessibility.
Further, on determining public accessibility the court went on to reject the
appellant‘s argument that a single catalogued thesis in one university library
does not constitute sufficient accessibility to those interested in the art
exercising reasonable diligence.

19. Merrell Dow Pharmaceuticals Inc. and Another v. H.N. Norton & Co.
Ltd. and Others, [1996] R.P.C. 76 HL
Merrell Dow discovered and patented an anti-histamine drug called
terfenadine. It is used by people who suffer from hay fever and similar allergies
and has the advantage that, unlike some other anti-histamines, it does not have
the side-effect of making one drowsy. In 1972 Merrell Dow obtained a patent
for terfenadine in the United Kingdom. After a period of extension under the
Patents Act 1977, it finally expired in December 1992. Other pharmaceutical
companies then started to make and market terfenadine.
In these proceedings Merrell Dow claimed that their monopoly in terfenadine
continues by virtue of a later patent (on terfenadine) which still has another 5
years to run. It was obtained in the following circumstances. After they had
patented terfenadine, they did some research into the way it worked. They found
that it passed through the stomach to be absorbed in the small intestine and was
then 99.5% metabolised in the liver. This was why it had no side-effects. They
analysed the chemical composition of the acid metabolite formed in the liver.
No one had identified it before. So they patented the acid metabolite as claim 24

147
of a patent granted in 1980 for a number of related anti-histamine products. This
is the patent in suit.
Therefore, Merell Dow argued that on manufacture and consumption of the
known anti-histamine drug terfenadine, the acid metabolite was necessarily
formed in the human body. Since, they owned patent on the acid metabolite,
use, sale and marketing of the drugs which produced the metabolite was an
infringement of the patent under section 60(2) of the Patents Act 1977.
On appeal to the House of Lords, the sole issue was whether, so far as
the claim to the metabolite included its manufacture by the action of
terfenadine in the human body, the patent in suit was invalid because the
invention was not new.
The appellants argued that the Patents Act 1977 had overturned previous
principles of United Kingdom patent law, namely that the Crown could not
grant a patent which would enable the patentee to stop another trader from
doing what he had done before, and that the test for anticipation before the
priority date was co-extensive with the test for infringement afterwards. Except
as provided by section 64, it was not enough that something should have been
done before; under the definition of the state of the art in section 2(2)
information about what was being done must have been made available to the
public. No information about the metabolite had been made available to the
public before the priority date of the patent in suit: no one could know about
something which he did not know existed.
The respondents said that although the metabolite had not previously been
identified chemically, its manufacture in the body was nevertheless part of the
state of the art.
Held, dismissing the appeal:-
(1) Article 54 of the EPC made it clear that to be part of the state of the art,
the "invention" must have been made available to the public. An invention was
a piece of information. Making matter available to the public within the
meaning of section 2(2) therefore required the communication of information.
The use of a product made the invention part of the state of the art only so
far as that use made available the necessary information. Under the 1977
Act a patent could be used, subject to section 64, to stop someone doing
what he had done before if the previous use was secret or uninformative.
Acts done secretly or without knowledge of the relevant facts, which would
amount to infringements after the grant of the patent, would not count as
anticipations before.
(2) The same thing might be known under one description and not known
under another. Section 2(2) did not purport to confine the state of the art about
products to knowledge of their chemical composition. It was part of the state
of the art if the information which had been disclosed enabled the public to
know the product under a description sufficient to work the invention. For
some purposes, at least, products need not be known under their chemical
description in order to be part of the state of the art. If a recipe which inevitably

148
produced a substance was part of the state of the art, so was that substance as
made by that recipe. Whether or not a person was working a product invention
was an objective fact independent of what he knew or thought about what he
was doing.
(3) In this case anticipation by use was similar to anticipation by disclosure
in that no one was aware that the product was being made. The distinction here
was that as regards the anticipation by use the acts relied upon conveyed no
information which would have enabled anyone to work the invention but relied
solely on the fact that the product had been made, while the anticipation by
disclosure relied upon the communication to the public of information which
enabled it to do an act having the inevitable consequence of making the product.
(4) In this case the "invention" was the making of the metabolite within the
human body by the ingestion of terfenadine
(5) Knowledge of the metabolite had been made available to the public by
the terfenadine specification. For many purposes the description was not
sufficient to make the product part of the state of the art. It would not enable
anyone to work the invention in the form of isolating or synthesizing the
metabolite. But it enabled the public to work the invention by making the
metabolite in their livers. The fact that they would not have been able to
describe the chemical reaction in these terms did not mean that they were not
working the invention. For the purposes of the particular invention in issue, the
terfenadine specification contained sufficient information about the metabolite
to make it part of the state of the art.
(6) If anticipation by use was to be treated separately from anticipation by
disclosure, it must be assumed that the participants in the clinical trials took
terfenadine without being at liberty to discover its composition and produced
within themselves a substance which had not then been readily capable of being
identified but was now known to have been the metabolite. Under the Patents
Act 1949 such uninformative use would undoubtedly have invalidated the
patent, but under the 1977 Act the claim for the metabolite could not be
dismissed simply on the ground that making it was something that had been
done before.

D. UTILITY
20. Brenner v. Mason, [38 US 519.]
In this case the inventor Mason had filed a patent application claiming a
method of making a steroid compound. Although the particular application
claiming a compound Mason was concerned with was already known to the art,
chemists were yet to identify any setting in which it could be gainfully
employed. However, as skilled artisans knew that another steroid with a very
similar structure had tumor inhibiting effects in mice, Mason‘s new method of
making the compound was a research to the scientific community. The PTO
Board of Appeals affirmed the examiner‘s rejection of the application. The
Board reasoned that because Mason could not identify a single use for the

149
steroid he had produced, the utility requirement was not satisfied. The board
was unimpressed by the argument that a similar compound had beneficial
effects. The matter went on appeal to the CCPA, wherein the CCPA held that
because the claimed process worked to produce a compound, the utility
requirement was satisfied.
The matter went to the US Supreme Court, where the decision was once
more reversed. The court held that an invention may not be patentable until it
has been developed to a point where “specific benefit exists in the currently
available form.”
―Until the process claim has been reduced to production of a product shown
to be useful, the metes and bounds of that monopoly are not capable of precise
delineation... Such a patent, if granted, may confer power to block whole areas
of scientific development, without compensating benefit to the public.
Thus, the requirement that a chemical process be useful is not satisfied by a
showing that the compound yielded belongs to a class of compounds which
scientists are screening for possible uses. Nor is the utility requirement for
chemical processes satisfied by a showing that the process works, i. e., yields the
intended product.
The court closed by noting that a patent is not a hunting license. It is not a
reward for the search, but a compensation for its successful completion.
52 . C. P. A. (Pat.) 739, 333 F.2d 234,
reversed.

21 Eli Lilly & Co v Human Genome Sciences Inc, (UK August 4, 2008).
A leading UK judge has clarified the position of the UK courts on the scope
of biotech patents and in particular gene sequence patents In his July 31
judgment in Eli Lilly & Co v Human Genome Sciences Inc, High Court judge
Mr Justice Kitchin laid down a set of principles which he said the UK courts
should apply when considering whether or not a given patent possessed
industrial applicability.
Kitchin said:
1) The notion of ‘industry’ should be construed broadly;
2) the capability of industrial exploitation must be derivable from the
description in the patent read in conjunction with the skilled person‘s common
general knowledge;
3) The description must disclose a practical way of exploiting the invention
in at least one field of industrial activity;
4) Conversely, the requirement will not be satisfied if what is described is
merely an interesting research result. The patent should not act so as to reserve
for the applicant an unexplored field of research;
5) If a substance is disclosed and its function is essential for human health
then the identification of the substance having that function will immediately
suggest a practical application, but if the function is not known or understood

150
and no disease has been identified which is attributable to an excess or a
deficiency of it then the requirement of industrial application is not satisfied;
6) Using the claimed invention to find out more of its own activities is not
itself an industrial application; and
7) It is no bar to patentability that the invention has been found by homology
studies using bioinformatics.
The case is thought to be the first in which the UK courts have had to
consider the circumstances in which a patent relating to a gene sequence can
validly be granted on the basis of it being capable of industrial application.
While the issue is not usually a problem in sectors such as the mechanical
arena, where a claimed invention frequently has a real and practical application,
the position has been less clear-cut in biotechnology, where there has been a
dearth of UK cases.
Justice Kitchin held that the Neutrokine-α patent filed by Maryland
biopharmaceutical company Human Genome Sciences in 1996 and granted on
17 August 2005 - was invalid on the grounds of obviousness, insufficiency and
lack of industrial applicability.
Neutrokine-α is one of a number of names given to a particular gene member
of the Tumour Necrosis Factor (TNF) super family of proteins. HGS was the
first of several companies to file for a patent on the TNF gene. But according to
Indianapolis-based pharmaceutical manufacturer Lilly, HGS did so without
having any real knowledge as to the biological activity or function of
Neutrokine-α, the identity of any receptor, the conditions which it causes or the
diseases which it could treat.
Justice Kitchin agreed with Lilly‘s revocation action, which claimed that
HGS‘ description of the activities and uses of Neutrokine-α had no scientific
data to support it and was therefore really a prediction based on knowledge of
other members of the TNF superfamily.
“In my judgment the skilled person would come to the conclusion that the
inventors had no idea as to the activity of Neutrokine-α when drafting the
patent,‖ said Kitchin. ―It teaches the skilled person nothing useful about its
activity other than that Neutrokine-α is another member of the TNF ligand
superfamily.”
HGS patent was filed before any traditional so-called wet experimental work
could be performed. It was only subsequent research that had shown that
proteins and antibodies described in the patent were of therapeutic use.
HGS subsequently developed a monoclonal antibody to Neutrokine-α which
is in clinical trials. But Lilly has spent $50 million developing its own
monoclonal antibody to Neutrokine-α and planned to spend another $250
million bringing it to the clinic.
Kitchin‘s ruling is a reminder that the state of the art when the patent‘s
validity comes under scrutiny cannot be used retrospectively in order to correct
deficiencies in the patent at the time when it was filed.

151
E. NON-OBVIOUSNESS
21. Graham v. John Deere, 383 U.S. 1.
Graham was concerned with the construction of a plow that would not break
upon striking the soil obstructions such as rocks. In 1950, he obtained the ‗811
patent, directed towards a plow with a spring clamp. The device featured an
upper plate secured to the lower flange of the H-beam of a plow frame. A hinge
plate was pivoted to the upper plate, with one end of the plow shank resiliently
and frictionally held between the upper and hinge plates. An opening in the
shank allowed a rod to extend through it. A coil spring was around the rod and
seated on the upper plate. When the plow hit an obstruction, the shank moved
downward against the tension of the coil spring and pivoted the hinge plate.
This pivoting action allowed the plow to pass over obstructions and increased
the resiliency of the shank. In 1950, Graham modified the ‗811 structure and
filed for a patent. That patent, granted in 1953, was the one litigated upon in the
present suit. The invention differed from the earlier invention in two ways:
(1) the ‗798 invention featured a stirrup and bolted connection of the shank
to the hinge plate, and (2) the ‗798 invention placed the shank below the hinge
plate. These changes permitted the plow shanks to be pushed upward when they
struck obstructions in soil and to spring back into normal position when
obstruction was passed over.
Subsequently, in an infringement suit, the Defendants challenged the validity
of the ‗798 Graham patent on the ground of obviousness.
Thus, in this case the US Supreme Court dealt with the parameter of
obviousness and reiterated the parameters laid down in the doorknob case. Clark
J. delivering the opinion of the court stated that, ―innovation, advancement,
and things which add to the sum of useful knowledge are inherent requisites in
patent system which by constitutional command must promote progress of
useful arts; such is the standard expressed in the Constitution and it may not be
ignored.
In light of this background the court enunciated the Graham test, wherein to
determine obviousness one had to follow the four step test. As per this four step
test, first, the court was to examine the scope and content of prior art. Second,
differences between prior art and claims at issue were to be ascertained. Third,
the level of ordinary skill in pertinent art is to be resolved and lastly, against this
background obviousness or non-obviousness of subject matter is determined.
Along with the four step test the Apex Court also laid out a set of exemplary
secondary considerations that could be used to negate a prima facie case of
obviousness, some of these secondary considerations being commercial success,
long felt but unsolved needs, failure of others, etc.
Applying these factors to the invention at hand the court concluded that the
patent is invalid under s.103 and, therefore, and affirmed the judgment of the
Eighth Circuit.

22. US v. Adams, 383 U.S. 39

152
This case was an action against government for patent infringement and for
breach of implied contract to pay compensation for use of invention. In this case
the patent under consideration was issued in 1943, it related to a battery
compromising of two electrodes – one made of magnesium and the other of
cuprous chloride – which were placed in a container. The electrolyte, or battery
fluid, used may be either plain or salt water. This combination of magnesium
and cuprous chloride never existed before in a single battery.
The Government contended that wet batteries comprising a zinc anode and
silver chloride cathode are old in the art; and that the prior art shows that
magnesium may be substituted for zinc and cuprous chloride for silver chloride.
Hence, it argued that the ‘combination of magnesium and cuprous chloride in
the Adams battery was not patentable because it represented either no change or
an insignificant change as compared the prior battery designs’.
The court disagreed with the claims of the government and stated that,
―Here, however, the Adams battery is shown to embrace elements having an
interdependent functional relationship. It begs the question, and overlooks the
holding of the Commissioner and the Court of Claims, to state merely that
magnesium and cuprous chloride were individually known battery components.
If such a combination is novel, the issue is whether bringing them together
as taught by Adams was obvious in the light of the prior art.
Thus, as regards obviousness the court held that, ―We conclude the Adams
battery was also nonobvious. As we have seen, the operating characteristics of
the Adams battery have been shown to have been unexpected and to have far
surpassed then-existing wet batteries. Despite the fact that each of the elements
of the Adams battery was well known in the prior art, to combine them as did
Adams required that a person reasonably skilled in the prior art must ignore that
(1) batteries which continued to operate on an open circuit and which heated in
normal use were not practical; and (2) water-activated batteries were successful
only when combined with electrolytes detrimental to the use of magnesium.
[For instance, in the Skrivanoff patent depended upon bu the Government to
prove obviousness, though the patent disclosed the use of magnesium in an
electrolyte, the disclosed use completely different from that used in Adams and
also practice had shown that the Skrivanoff formulation was both dangerous and
inoperable.] These long-accepted factors, when taken together, would, we
believe, deter any investigation into such a combination as is used by Adams.
This is not to say that one who merely finds new uses for old inventions by
shutting his eyes to their prior disadvantages thereby discovers a patentable
innovation. We do say, however, that known disadvantages in old devices
which would naturally discourage the search for new inventions may be taken
into account in determining obviousness.‖
―Nor are these the only factors bearing on the question of obviousness. We
have seen that at the time Adams perfected his invention noted experts
expressed disbelief in it. Several of the same experts subsequently recognized
the significance of the Adams invention, some even patenting improvements on

153
the same system. Fischbach et al., U.S. Patent No. 2,636,060 (1953).
Furthermore, in a crowded art replete with a century and a half of advancement,
the Patent Office found not one reference to cite against the Adams application.‖
On this ground Adams patent was held to be novel, non-obvious and valid.

23. KSR v. Teleflex, 127 S.Ct. 1727.


The KSR International case provided the Supreme Court with its first
opportunity to review the TSM test created and promulgated by the Federal
Circuit and the CCPA.
Teleflex, Inc. (―Teleflex‖) sued KSR International Co. (―KSR‖) in the U.S.
District Court for the Eastern District of Michigan for infringement of a patent
over which Teleflex held an exclusive license. The patent (―565‖) disclosed a
position-adjustable vehicle pedal assembly that Teleflex accused KSR of
making. KSR then filed a Motion for Summary Judgment on the invalidity of
the '565 patent. The District Court disposed of the case by granting KSR's
Motion for Summary Judgment on the invalidity of the '565 patent on the
ground that the '565 patent was obvious based on these findings under the TSM
test. Teleflex appealed the District Court's decision to the Federal Circuit
complaining that the district court used a ―incorrect teaching-suggestion-
motivation test‖ in its decision. The Federal Circuit reviewed the facts of the
case de novo, reapplying the TSM test. In its application, the court agreed with
Teleflex that the district court ―did not apply the correct teaching-suggestion-
motivation test.
It is pertinent to note that the District Court held that the motivation to
combine was deemed necessary under the test for combining the ―pedal
assembly in which the pivot does not move with pedal adjustment‖ with the
―modular pedal position sensors‖ and the same was evident from the
knowledge that a person with ordinary skill in the art would have of the
applicable prior art. Moreover, the court also found ―express teachings with
respect to the desirability‖ of combining a pedal assembly with a sensor in prior
art. Hence, revoking the patent granted on the ground of obviousness.
The Federal Circuit did not agree with the District Court's application of the
test because, according to the Federal Circuit, the TSM test requires the
court to ―make specific findings as to whether there was a suggestion or
motivation‖ to combine the prior art and there is no space for common
knowledge. The court found that the two pieces of prior art the District Court
combined were not addressing the same problem. Therefore, according to the
Federal Court, the District Court could not have used the nature of the problem
to be solved as the motivation to combine. The Federal Circuit also took issue
with the express teachings the District Court found in another piece of prior art
because that piece of prior art did not relate to adjustable pedal assemblies but
to wire chafing in pedal assemblies. Hence, the court concluded that there was
nothing that would result in a PHOSITA cross referencing form one claim to
another. Thus, the Federal Circuit vacated the grant of summary judgment in

154
favor of KSR and remanded to the district court for a new determination under
the teaching-suggestion-motivation test on January 6, 2005.
KSR then petitioned the Supreme Court for a writ of certiorari on April 6,
2005. The Supreme Court granted certiorari to hear KSR International Co. v.
Teleflex, Inc. on June 26, 2006. Before the Supreme Court, after hearing the
arguments, Kennedy J., delivering the decision on behalf of the Apex Court
agreed with the decision of the District Court and held that the patent was
invalid as it was obvious. More importantly, the Apex Court critiqued the rigid
application of the TSM test by the Federal Circuit and stated that, ―in
determining whether subject matter of patent claim is obvious, neither the
particular motivation nor the avowed purpose of patentee controls; what matters
is the objective reach of the claim.‖ It also stated that while the TSM test can
only be used as useful indicia to help in determining such objective reach, the
Graham test remains the primary test of obviousness of a claimed invention.
On reviewing the TSM test, the Supreme Court analysed the manner in
which the Federal Court applied the TSM test and decided the KSR
International decision as given by the Federal Court was a textbook example of
rigid application of the TSM test and the problems that the same can create.
The Apex Court severally critiqued the decision of the Federal Court, and
stated that Graham‟s case had laid down an ‗expansive‘ and ‗flexible‘
approach to the obviousness question, and the rigid application of the TSM test
is inconsistent with the same. Addressing the Federal Circuit‘s assertion that on
correct application of the TSM test, prior art references from other areas (those
other than the area of claimed invention) cannot be adopted in order to prove
obviousness of a claimed invention in a given field, as no teaching, suggestion
or motivation teaches, motivates or suggests combination of such references
from varying fields when an individual is addressing a specific problem in a
specific field, the Apex Court stated that such rigid application of the TSM test
is wrong because when a work is available in one field, design incentives and
other market forces can prompt variations of it, either in the same field or in
another and a PHOSITA has sufficient creativity to recognize such variations
and utility of the same.
Moreover, the Apex Court also held that with the creative background of the
PHOSITA and allowance of mosaicing prior art references and by drawing
inferences from the same, one need not seek out precise teachings directed to
the challenged claim‘s specific subject matter in order to prove obviousness.
Further, clearly rejecting the Federal Circuit‘s approach, the Apex Court also
cautioned against placing overemphasized importance on published articles and
the explicit content of issued patents.
However, it is pertinent to note that though the Apex Court critiqued the rigid
application of the TSM test by the Federal court, in principle it accepted the
TSM test within the jurisprudence of the obviousness parameter. The court
stated that the TSM test captures a helpful insight, especially with regard to
combination patents. This helpful insight made by application of the test was to

155
an extent evident from Adams case, where the fact that prior art taught away
from the claimed invention clearly proved that the invention was non obvious.
Also, another parameter of the TSM test, i.e., the need or problem known in
any field being a motivation to combine prior art references in the manner
claimed is helpful, as it helps prove obviousness in the face of a finite number
of known solutions to the given problem.
Having accepted the test, the court attempted to give guidelines regarding its
application such that it would be in consonance with the test laid down in
Graham‟s case. The Court stated that the TSM test was to be restricted to be
treated as a helpful insight in application of the Graham‟s test and cannot be
made the primary test to decide on the obviousness of a claimed invention. This
statement was based on the reasoning that, ―the diversity of inventive pursuits
and of modern technology counsels against limiting the analysis only to the
TSM test. In many fields there may be little discussion of obvious techniques or
combinations, and it often may be the case that market demand, rather than
scientific literature, will drive design trends. Sole application of the TSM test, in
a rigid manner, would result in grant of patent protection to such advances
which are without real innovation resulting in retardation of progress and in
case of patents combining previously known elements, deprivation of value or
utility of prior inventions.‖ Thus, the Court's decision confirmed that the TSM
test remains good law, but the ruling clarified that ―an expansive and flexible‖
type of approach is to be used when applying the test.

24. Windsurfing International v. Tabur Marine (1985) RPC 59,CA


Before: Lord Justice Oliver Lord Justice O'Connor and Lord Justice Slade
A patent had been granted for the novelty of a free sailing rig which could be
fitted to any sort of wind propelled vehicle. The patentee filed an infringement
suit against the Defendant, who counter claimed on two grounds, one that there
was prior use by others, hence, the invention lacked novelty, and secondly that
it was obvious from a prior printed publication.
The prior art cited for the purposes of novelty was a prior use that had taken
place some ten years before the date of the patent when C (then a 12-year-old
boy) had built a sailboard and used it on an inlet near Hayling Island on summer
weekends during two consecutive seasons. The user was open and visible to
anyone in the vicinity of the caravan site where the family stayed. C's sailboard
differred from what was claimed only in that the sail was held between a pair of
booms which were straight when the vehicle was at rest. However the evidence
was that in use the booms were sufficiently flexible to assume an arcuate shape
Patents Court, Whitford J. found the claims invalid as anticipated by the prior
user and obvious but he made no finding of obviousness based on the prior user
The plaintiffs then sought to amend by restricting their claims specifically to
a surfboard of the Californian type as shown in Figure 1 of the drawings, but the
judge concluded that there was no scope for such an amendment and refused to
allow it.

156
The plaintiffs appealed to the Court of Appeal against the findings of
anticipation and obviousness
Held, to be lacking inventive step, dismissing the appeal and upholding the
defendants' contentions on the respondents' notice:
As regards determining non–obviousness, the court stated that such
determination was dependent on answering the required four steps:
i. Identifying the inventive concept embodied in the patent;
ii. Imputing to a normally skilled but unimaginative
addressee what was common general knowledge in the art
at the priority date;
iii. Identifying the differences if any between the matter cited
and the alleged invention; and
iv. Deciding whether those differences, viewed without any
knowledge of the alleged invention, constituted steps
which would have been obvious to the skilled man or
whether they required any degree of invention.
The inventive concept of the patent in suit was the free-sail concept. At the
priority date anyone familiar with sailing craft would have known as part of his
general knowledge the difference between square and Bermuda rigs and the
disadvantages of the former for manoeuvrability. The printed article disclosed
the same inventive concept and the only difference of substance was the use of a
square rig held by a crossed spar and boom instead of a Bermuda rig held by a
wishbone boom. One did not have to assume that the skilled man would treat
the article as something which had a great commercial future which he must
bend every effort to develop and improve, but he must at least be assumed to
appreciate and understand the free-sail concept taught by the article and to
consider in the light of his knowledge and experience how the vehicle described
would work on the water. In the light of the evidence, the skilled man would
have immediately recognized the disadvantages of the square rig in D, and it
would have been immediately obvious to him by applying his own general
knowledge that these disadvantages would disappear if the rig were changed to
Bermuda. It would also have been obvious to him that this change required the
sail to be stretched by means of a wishbone boom
Any skilled adult who applied his mind to C's device would at once have
seen it as obvious that the unconventional and primitive split boom ought to be
replaced by a wishbone boom.
Anticipation by the prior user was established. A skilled man reading the
specification would at once identify what was described as "arcuate booms" as a
wishbone boom and C used what was clearly a primitive form of wishbone boom, C's
device would quite clearly have constituted an infringement of the patent if it had
been manufactured in 1980 instead of 1958.

157
25. Conor Medsystems v. Angiotch Pharmaceuticals Inc. House of Lords (July 11,
2008)

In this case the House of Lords upheld the validity of Angiotech‘s European patent
for a paclitaxel eluting stent and reversing the decisions of the UK court of First
Instance and the CAFC. The CAFC held the patent invalid on the ground that the patent
was invalid cause it contained insufficient experimental data.

As regards the concept of obviousness, Lord Hoffman rejected that the test of
whether an invention has a reasonable expectation of success to determine obviousness.
According to the law lord, the concept of expectation of reasonable success was an
oxymoronic concept, with no precedent in the law of patents.

Clarifying the test of obviousness, Lord Hoffman stated that, ―In my


opinion, the invention is the product specified in the claim and not some vague
paraphrase based upon the extent of his disclosure in the description. Thus, if a
specification passes the threshold tests of disclosing enough to make the
invention plausible there should be no reason why the question of obviousness
should be subject to a different test according to the amount of evidence which
the patentee presents to justify a conclusion that his patent would work.”

OWNERSHIP, RIGHTS & INFRINGEMENT OF THE RIGHTS


A. Literal Infringement
26. Markman v. Westview Instruments No. 95-26, 64 U.S.L.W. 4263 (1996)
[SC]
A patent is infringed literally if the accused product or process includes
every element exactly as recited in at least one of its claims. Standard of
‘absolute identity’ is applied.
In this case the patented invention at issue was an inventory for tracking
system for a dry cleaning shop. Markman combined computer and bar code
technology to minimize lost garments and employee theft during the dry
cleaning process. Thus, it was a system that can monitor and report the status,
location, and movement of clothing in a dry -cleaning establishment. His
claimed ‘inventory control and reporting system’ allowed the detection of
spurious additions to the inventory as well as spurious deletions therefrom.
Markman‘s assertion of the patent against a competitor led to a jury trial.
The defendant‘s system tracked invoice numbers, total prices, and receivables.
However, it did not track clothing. At issue was the meaning of the word
“inventory” in the patent claims, whether inventory means: clothing
and/or totals. Markman argued that the inventory included clothing and/or
totals, while the defendant argued that Inventory meant clothing. Although the
jury found for Markman, the Trial Judge instead directed a decision of non-
infringement on the basis that the term ‘inventory’ referred exclusively to
articles of clothing. Because the accused device merely maintained a listing of

158
invoices, it could not track the location of individual garments as they moved
about the shop and therefore could not infringe.
On appeal, the Federal Circuit affirmed. In so doing, the court established
two categories of evidentiary inputs for use in claim interpretation. The first, so
called ‘intrinsic evidence’, consisted of the claims, the specification, and the
prosecution history. Courts were required to consider all the intrinsic evidence
on record to determine the meaning of the claims. All other sources, ranging
from dictionaries to expert testimony, was classified as ‘extrinsic evidence’.
The Court also held that interpretation was a matter of law, solely for the judge
to decide.
The matter went up to the Supreme Court. The Apex Court affirmed the
dicta of the Federal Court.

B. PITH AND MARROW DOCTRINE – U.K.


27. Catnic Components v. Hill & Smith [1982] R.P.C. 183 H.L.
In this case the claim of the patent was for a lintel, used to provide structural
support over a door or window opening in a brick wall, the structure of the lintel
was such that the upper plate of the same was supported upon the lower plate by
two rigid supports, one in the front and the other ‘extending vertically’ from the
one plate to the other at the rear. The Defendant‘s lintel had a rear support
which was inclined 6 degrees or 8 degrees from the vertical. The Defendant
argued that defendant‘s lintel would have to be exactly vertical to infringe.
The House of Lords decided that this variation had no material effect upon
the load bearing capacity of the lintel or the way it worked, and that this would
have been obvious to the skilled builder at the date of publication of the patent.
It also decided that the skilled reader would not have understood from the
language of the claim that the patentee was insisting upon precisely 90 degrees
as an essential requirement of his invention. The conclusion was that ‘extending
vertically’ meant extending with the range of angles which give substantially the
maximum load-bearing capacity and of which 90 degrees is the perfect example.
Thus, it is important to note that Lord Diplock adopted purposive
construction. Thus, it is pertinent to note that the Catnic principles are
applicable when a defendant‘s device falls outside the literal scope of the claim.
While constructing a claim and determining infringement, the court should
consider the essential features of the invention (―pith and marrow doctrine‖)
and then by adopting purposive interpretation, see whether the variants are
similar to the original aspects of the invention.
While applying this purposive construction, the House of Lords laid down
three-step test:
1. Does the variant have a material effect upon the way the invention works?
If yes, the variant is outside the patent claim
If no, the next question comes
2. Would the fact that the variant had not material effect have been obvious
at the date of publication of the patent to a reader skilled in the art?

159
If no, the variant is outside the claim
If yes, the third question needs to be asked
3. Would the reader skilled in the art nevertheless have understood from the
language of the claim that the patentee intended that strict compliance with the
primary meaning was an essential requirement of the invention
If yes, the variant is outside the claim
If no, then there is infringement.

28. Improver Corp. v. Remington Consumer Products Ltd. [1990] F.S.R. 181
In this case, the Court of Appeal emphasized that the Catnic principles are
applicable when a defendant's device falls outside the literal scope of the claim.
The court stated that the Catnic principles are consistent with Article 69 of
European Patent Convention and the protocol on interpretation of Article 69.
In the case the patent covered a depilatory device having a helical spring driven
by a motor. The spring when rotated gripped hairs between its coils and plucked
them from the skin. The alleged infringement replaced the spring with a rubber rod
having slits at the surface. The question was whether the slit rubber rod was a
―helical spring‖.
Dealing with the issue of equivalents as a guide to claim construction, the court
that,
--if the issue was whether a feature embodied in an alleged infringement which
fell outside the primary, literal or a contextual meaning of a descriptive word or
phrase in the claim ('a variant') was nevertheless within the language as properly
interpreted, the court should ask itself the following three questions:
(1) Does the variant have a material effect upon the way the invention works? If
yes, the variant is outside the claim. If no
(2) Would this (i.e. that the variant had no material effect) have been obvious at
the date of publication of the patent to a reader skilled in the art? If no, the variant
is outside the claim. If yes--
(3) Would the reader skilled in the art nevertheless have understood from the
language of the claim that the patentee intended that strict compliance with the
primary meaning was an essential requirement of the invention? If yes, the variant
is outside the claim.
On the other hand, a negative answer to the last question would lead to the
conclusion that the patentee was intending the word or phrase to have not a literal
but a figurative meaning denoting a class of things which include the variant and
the literal meaning, the latter being perhaps the most perfect, best known or
striking example of the class.”
Applying the questions to the facts of the case the court held that to a PHOSITA
on reading the claim it would show that the patentee intended strict compliance
with the component ―helical spring‖ in its primary meaning and not in a wide
generic sense, as an essential feature of the invention. Hence, on this basis the
court held that the rubber rod fell outside the meaning of ‘helical spring’ and did
not infringe.

160
29. Raj Parkash vs Mangat Ram Chowdhry And Ors. AIR 1978 Delhi 1

Facts:

The plaintiff/appellant produces and markets a toy called a viewer. This viewer


uses a 35 mm. medially cut positive film on which are printed one or more pictures
viewed through a lens fitted in a viewer specially adapted for use of such medially
cm strips of 35 mm. positive films so as to get a virtual image of the same size as
the virtual image of a normal frame in a 35 mm. film.It is he plaintiff's case that the
defendants have infringed the patent of the plaintiff inasmuch as they are
manufacturing and selling in the open market film strip viewers utilising a
medially cut 35 mm. cinematograph film in viewers in a manner which is identical
to and a copy of the viewers being manufactured and marketed by the plaintiff. 

Issue:

Whether the invention of the Plaintiff is patentable and therefore, the Defendants
have infringed the plaintiff’s patent?

Judgement:

The Delhi High Court disposed the appeal upholding the validity of the patent of
the plaintiff, and issuing an injunction restraining the defendants from in any way
infringing the said patent of the plaintiff during its turn or extension thereof. 

Proposition:

 Invention, as is well-known, is to find out something or discover something


not found or discovered by anyone before, determined by a true and fair
construction of the specifications.
 Heald K. C. in Electrical and Musical Industries, Ld. and Boonton Research
Corporation Ld. v. Lisson Ld. and another, 1937 (54) R.P.C. 307(1) and
affirmed by Romer, J. in the same case :- "As in all patent actions in which the
validity of the patent is put in issue, the first thing to be determined is the
proper construction of the specification ; but the specification must be
construed with a proper appreciation of the meaning that it would convey to
persons engaged in the art to which it relates, for it is to those persons that it is
addressed and it must be read with their eyes."
 It is the pith and marrow of the invention claimed that has to be looked into
and not get bogged down or involved in the detailed specifications and claims
made by the partics who claim to be patentee or alleged violaters. (See
Birmingham Sound Reproducers Ld. v. Collaro Ld. and Collaro Ld. v.
Birmingham Sound Reproducers Ld.. 1956 R.P.C. 232)(2).
 To quote from another well-known decision of the Court of Appeal in R.C.A.
Photo- phone Ld. v. Gaumont-British Picture Corporation Ld. and British
Acoustic Films Ltd., 1936 R.P.C. 167(4). 'The specification must be construed
in the first instance as a written instrument and without regard to the alleged
infringement: ....".

161
 It is settled law that the title of the specifications of an invention claimed does
not control the actual claim. 
 In Osram Lamp Works Ld. v. Pope's Electric Lamp Company Ld., 1915
R.P.C. 538(6) the above aspect is further highlighted by Joyce, J. who
emphasised that the essential part of the specifications have to be carefully
read and held as an invention, if novel, without getting mixed up in the
unessential and the usual parts of processes which are well-known. 
 The process of photography mentioned in the specifications and the claims, in
our view, unduly prolific, does not detract from the essential features of the
plaintiff's invention of which, as has already been held by us, there is neither
anticipation nor prior user.
 Unessential features in an infringing article or process are of no account. If the
infringing goods are made with the same object in view which is attained by
the patented article, then a minor variation does not mean that there is no
piracy. A person is guilty of infringement if he makes what is in substance the
equivalent of the patented article. Some trifling or unessential variation has to
be ignored.
 Lord Denning, M. R. in Beecham Group Limited v. Bristol Laboratories Ltd.
and another, 1967 (16) R.P.C. 406 (12) :- “A person is guilty of infringement
if he makes what is in substance the equivalent of the patented article. He
cannot get out of it by some trifling or unessential variation.”
 The defendants have made certain variations in its viewers but these are
unessential; and what the defendants market is substantially the same thing, as
was conceived by the plaintiff.

30. CTR Manufacturing Industries Limited, Pune v Sergi Transformer


Explosion Prevention Technologies Private Limited, Haryana and others,
2015 Indlaw MUM 1617

Facts:

The Plaintiff ("CTR") says it has a patent for "A System And Method For
Preventing and/or Detecting Explosion And/Or Fire Of Electrical Transformers".
CTR claims that Defendant Sergi's product, the SERGI 3000, is an infringement.
The patent to which CTR lays claim, Sergi says, was incapable of being granted:
everything it claims was known in the prior art, including by CTR's own
publications. Sergi also says that CTR's plaint is deliberately deceptive and
contains crucial omissions. CTR says that the essential integers of Sergi's product
are the same as those in CTR's Patent.

Issue:

Whether the claimed invention is patentable and hence infringed by the defendant?

Judgment:

The Bombay High Court allowed the appeal by holding that the PRV is not
essential, either to the CTR patent or to Sergi's product or the DTL tender; the

162
addition of the PRV is a non-essential addition; the pith and marrow of CTR's
invention is the two integers working interdependently to produce what is, prima
facie, a dramatically improved result at the stage of a explosion detection (not fire
extinguishing or fire detection and prevention); Sergi has entirely abandoned its
patent and has adopted CTR's; Sergi could not have met the DTL bid without such
an infringement.

Proposition

 Wadia J in Lallubhai Chakubhai v Chimanlal & Co., AIR 1936 Bom 99 1935
Indlaw MUM 89 said that sometimes a patent is infringed by the taking of
only a part; but that depends on whether part so lifted and for which protection
is sought is a new and material part, especially in the case of a combination. I
read the word 'part' in this phrasing not to mean an individual integer per se
but a portion of the patent. This has been the law for a very long time: if the
defendant really has taken the substance and essence of the plaintiff's
combination, its 'pith and marrow', the mere fact that certain parts are added or
omitted does not mean there is no infringement. Proctor v Bennis, [1886] 36
Ch. 740
 The law then evolved to Catnic Components Ltd. v. Hill & Smith Ltd., [1982]
RPC 183 : [1981] F.S.R. 60 in which Diplock LJ, borrowing quite openly
from doctrines of statutory interpretation, advocated a 'purposive' approach to
construction: “My Lords, a patent specification is a unilateral statement by the
patentee, in words of his own choosing, addressed to those likely to have a
practical interest in the subject matter of his invention (i.e. "skilled in the art"),
by which he informs them what he claims to be the essential features of the
new product or process for which the letters patent grant him a monopoly. It is
called "pith and marrow" of the claim. A patent specification should be given
a purposive construction rather than a purely literal one derived from applying
to it the kind of meticulous verbal analysis in which lawyers are too often
tempted by their training to indulge. The question in each case is: whether
persons with practical knowledge and experience of the kind of work in which
the invention was intended to be used, would understand that strict compliance
with a particular descriptive word or phrase appearing in a claim was intended
by the patentee to be an essential requirement of the invention so that any
variant would fall outside the monopoly claimed, even though it could have no
material effect upon the way the invention worked.”
 Read together, those claims make it clear that CTR has wholly eliminated and
therefore disclaimed the PRV. It insists on the two critical integers I have
mentioned earlier working together. These, working together, form the 'pith
and marrow' of CTR's Patent. Removing either of them cannot meet either the
DTL tender requirements or deliver the same result.
 Sergi claims the PRV to be essential to both its product and its patent. But if it
is essential it cannot possibly be sidelined with an "or" as is shown in Sergi
India's brochure and even in the comparative chart. 
 If a patent claim is overbroad, and for that reason, includes in the sweep of the
case for infringement the defendant's invention, i.e., by including every single
non-essential and essential addition, then the patent may be prima facie invalid
for want of novelty. No person can claim such a monopoly as would result in a
bar to future improvements. If the patent is, on the other hand, so narrow that

163
it excludes the addition of another essential integer, then there is no
infringement.

Test for Doctrine of Equivalents – U.S.


31. Graver Tank & Mfg. v. Linda Air Products Co. 339 U.S. 605 (1950)
The patent at issue in this case was an electric wielding composition termed a
―flux‖. When applied to the surfaces to be joined through welding, a flux
assists in the fusing of the two metals. The claimed flux consisted of calcium
fluoride and alkaline earth metal silicate. The accused flux was known under the
Trademark Lincolnweld 660. It consisted of silicates of calcium and manganese.
It is pertinent to note that manganese is not an alkaline earth metal. However,
the patentee also marketed a flux consisting of silicates of calcium and
magnesium, known as Unionmelt Grade 20. Also, prior art patents employed
manganese as a welding composition. Thus, while deciding the case the court
accounted for all the relevant patents and prior art.
While examining the infringement claim, the court stated that, “patent claims
must protect the inventor not only from those who produce devices falling
within the literal claims of the patent but also from copyists who make
unimportant and insubstantial changes and substitutions in the patent which,
though adding nothing, would be enough to take the copied matter outside the
claim, and hence outside the reach of law.”
As regards what would constitute equivalents, the court stated that when the
accused product or process performed substantially the same way to obtain the
same result, an infringement had occurred under the doctrine of equivalents.
The known interchangeability of the claimed and substituted ingredients
was an important factor in an equivalency determination. Such equivalency was
to be proven through the use of experts, learned texts and the disclosures in
prior art.
Applying these principles to the facts at hand the court affirmed the finding
of infringement under the doctrine of equivalents. As per the dicta of the court,
―the chemists had testified that magnesium and manganese were equivalents in
the welding art, and the trial court had properly found that the Lincolnweld and
Unionmelt fluxes were in all respects equivalent. It is also important to note that
the court saw no evidence that the Defendant had engaged in independent
development of the Lincolnweld flux, therefore, the court found it “difficult to
conceive of a case more appropriate for application of the doctrine of
equivalents.”
Hence, in this case the court adopted the so-called function-way-result test
in finding the infringement via the doctrine of equivalents. As is evident from
the case, the function-way-result test considered whether the element in the
accused device "does substantially the same function in substantially the same
way to achieve substantially the same result" as the element in the patented
claim.

164
32. Warner-Jenkinson v. Hilton Davis (516 U.S. (1996)
The petitioner and respondent both manufacture dyes from which impurities
must be removed. Respondent's ― '746 patent,‖ which issued in 1985, discloses
an improved purification process involving the ―ultrafiltration‖ of dye through
a porous membrane at pH levels between 6.0 and 9.0. The inventors so limited
their claim's pH element during patent prosecution after the patent examiner
objected because of a perceived overlap with the earlier ―Booth‖ patent, which
disclosed an ultrafiltration process operating at a pH above 9. 0.
In 1986, petitioner developed its own ultrafiltration process, which operated
at a pH level of 5.0. Respondent sued for infringement of the '746 patent,
relying solely on the ―doctrine of equivalents, under which a product or
process that does not literally infringe upon the express terms of a patent claim
may nonetheless be found to infringe if there is ―equivalence‖ between the
elements of the accused product or process and the claimed elements of the
patented invention.
The United States District Court for the Southern District of Ohio, Herman J.
Weber, J., entered judgment in favor of patentee, and competitor appealed. The
Court of Appeals of the Federal Circuit affirmed, and certiorari was granted.
While considering the matter, Justice Thomas on behalf of three judge bench
refused a number of arguments for the removal or restriction of the doctrine of
equivalents. The Apex Court held that even though an alleged infringer may not
fall within the literal wording of the claims, courts may find infringement when
there are "insubstantial differences" between the claim language and the
defendant's product or process
Further, the Apex Court also refused Warner-Jenkinson‘s argument that the
equivalents should be restricted to variants described in the patent‘s
specification or alternatively, to those variants known at the time the patent was
issued. The reasoning behind the rejection was that the perspective of the
PHOSITA is a part of the doctrine and has to be accounted for. Moreover, this
perspective in itself places sufficient limitation on misapplication of the
doctrine.
Further, the court clarified that intent was not an element in the analysis of
equivalency. Thus, even if the alleged infringer had not intended to create an
equivalent, if the doctrine was satisfied, he would still be liable for
infringement.
As regards the function-way-result test in enunciated in Graver Tank, the
Supreme Court stated that, ―the essential inquiry under doctrine of
equivalents is whether accused product or process contains element
identical or equivalent to each claimed element of patented invention.
However, the Apex Court also adopted for the reasoning of the Federal Circuit
as it accepted that the function-way-result test remains as one method for
determining equivalence, and particularly for mechanical devices, it is not the

165
only test. Rather, the court could also ―the substantiality of the differences‖
between the claimed and accused elements.
Next, the Supreme Court also considered the effect of prosecution history
estoppel on the doctrine of equivalents. Where no explanation for a narrowing
amendment had been provided, the court shifted the burden of the uncertainty
on to the patentee:
―Where no explanation is established, however, the court should
presume that the patent application had a substantial reason related to
patentability for including the limiting element added by amendment. In
those circumstances, prosecution history estoppel would bar the
application of the doctrine of equivalents as to that element.‖
However, the court also stated that, ―Where reason for change in
patent claim during prosecution was not related to avoiding prior art, or
otherwise to address a specific concern-such as obviousness, change may
introduce new element, but does not necessarily preclude infringement by
equivalents of that element.‖
By this means, the Supreme Court tried to strike a balance between the right
of the public to have a clear notice of the scope of the patent, and the right of the
patentee to protect its patent in terms of the doctrine of equivalents.
To reiterate, the court stated that, for the purposes of prosecution history
estoppel, ―There are a variety of reasons why the PTO may request a change in
claim language, and if the patent holder demonstrates that an amendment had a
purpose unrelated to patentability, a court must consider that purpose in order to
decide whether an estoppel is precluded. Where the patent holder is unable to
establish such a purpose, the court should presume that the purpose behind the
required amendment is such that prosecution history estoppel would apply.
Here, it is undisputed that the upper limit of 9.0 pH was added to the '746 patent
in order to distinguish the Booth patent, but the record before this Court does
not reveal the reason for adding the lower 6.0 pH limit. It is therefore
impossible to tell whether the latter reason could properly avoid an estoppel.‖
Hence, what is important to note is that the bar of prosecution history
estoppel is a flexible and rebuttable bar.
The court also made it clear that equivalency should be judged at the time of
infringement. This means that the equivalency may be extended so that an
element of an invention that was not an equivalent at the time of the invention
can later become an equivalent.
To reiterate, the Supreme Court, Justice Thomas, held that: (1) doctrine of
equivalents is not inconsistent with Patent Act; (2) doctrine of equivalents must
be applied to individual elements of patent claim, not to invention as a whole;
(3) prosecution history estoppel does not apply in every instance where the
patent claim has been amended during application process, regardless of reasons
for change; (4) patentee's addition of lower pH limit during application
process did necessarily preclude application of doctrine of equivalents as to
that element; (5) remand was required to determine whether reason for

166
amending patent claim to add lower pH limit was sufficient to avoid
prosecution history estoppel; (6) doctrine of equivalents does not require
proof of intent; (7) doctrine of equivalents is not limited to equivalents disclosed
within patent itself.

33. Festo Corp. v. Shoketsu Kinzoku Kogyo Kabushiki Co., Ltd. , 535 U.S.
722
Kennedy J. begins the opinion by stating that, ―This case requires us to
address once again the relation between two patent law concepts, the doctrine of
equivalents and the rule of prosecution history estoppel. The Court considered
the same concepts in Warner-Jenkinson Co. v. Hilton Davis Chemical Co.,
520 U.S. 17, 117 S.Ct. 1040, 137 L.Ed.2d 146 (1997), and reaffirmed that a
patent protects its holder against efforts of copyists to evade liability for
infringement by making only insubstantial changes to a patented invention. At
the same time, we appreciated that by extending protection beyond the literal
terms in a patent the doctrine of equivalents can create substantial uncertainty
about where the patent monopoly ends. If the range of equivalents is unclear,
competitors may be unable to determine what a permitted alternative to a
patented invention is and what is an infringing equivalent.
To reduce the uncertainty, Warner-Jenkinson acknowledged that competitors
may rely on the prosecution history, the public record of the patent proceedings.
In some cases the Patent and Trademark Office (PTO) may have rejected an
earlier version of the patent application on the ground that a claim does not meet
a statutory requirement for patentability. 35 U.S.C. § 132 (1994 ed., Supp. V).
When the patentee responds to the rejection by narrowing his claims, this
prosecution history estops him from later arguing that the subject matter
covered by the original, broader claim was nothing more than an equivalent.
Competitors may rely on the estoppel to ensure that their own devices will not
be found to infringe by equivalence.”
In the present case: the Federal Circuit held that by narrowing a claim to
obtain a patent, the patentee surrenders all equivalents to the amended claim
element. The petitioner argued that such an interpretation was a deviation from
the earlier cases, where PHE created a flexible bar, which applied only when
amendments were made to avoid prior art or earlier inventions. Thus, the
petitioner challenged the dicta of the Federal Circuit on the ground that by
replacing the flexible bar with a complete bar the Court of Appeals cast doubt
on many existing patents that were amended during the application process
when the law, as it then stood, did not apply so rigorous a standard.
The matter came before the Supreme Court which considered two important
questions in the decision:
• What kinds of claim amendment will cause prosecution history
estoppel?
• When there is estoppel, what range of equivalents will be available?

167
As regards the relationship between the two doctrines the court stated that,
―Prosecution history estoppel ensures that the doctrine of equivalents remains
tied to its underlying purpose. Where the original application once embraced the
purported equivalent but the patentee narrowed his claims to obtain the patent or
to protect its validity, the patentee cannot assert that he lacked the words to
describe the subject matter in question.
Going on to address the questions, the court ruled that any claim narrowing
amendments made to comply with the patent statute, not just amendments made
to prior art, will give rise to prosecution history estoppel. This means that
amendments to satisfy § 112 of 35 U.S.C. (rules relating to enablement, best
mode, adequate description) also could provoke prosecution history estoppel.
The court stated that, ―Estoppel arises when an amendment is made to secure
the patent and the amendment narrows the patent's scope. If a § 112 amendment
is truly cosmetic, then it would not narrow the patent's scope or raise an
estoppel. On the other hand, if a § 112 amendment is necessary and narrows the
patent's scope-even if only for the purpose of better description-estoppel may
apply. A patentee who narrows a claim as a condition for obtaining a patent
disavows his claim to the broader subject matter, whether the amendment was
made to avoid the prior art or to comply with § 112.
However, the court rejected the "complete bar" rule that had been adopted by
a majority of the Federal Circuit, and instead supported the "flexible bar" by
holding that the scope of an estoppel "requires an examination of the subject
matter surrendered by the narrowing amendment".
The Supreme Court also made it clear that since a patentee's decision to
narrow his claims through amendment may be presumed to issue a general
disclaimer of the territory between the original claim and the amended claim.
The patentee then has the burden of showing that the amendment that gave rise
to the estoppel "does not surrender the particular equivalent in question.
In order to rebut the presumption that the estoppel bars all equivalents, the
court reasoned that "the patentee must show that at the time of the amendment
one skilled in the art could not reasonably be expected to have drafted a claim
that would have literally encompassed the alleged equivalent".
The court identified three situations where an estoppel might not eliminate
all equivalents: the patentee can avoid the bar by providing that, at the time of
the amendment was made, the alleged equivalent was not foreseeable, or that
claiming or describing the equivalent would not have been possible or
reasonable.
In addition, the court stressed the notice function of claims, holding that
claims are supposed to give notice to others of what activities they can
permissibly engage in. It also explained that it would be unfair to change
routinely the boundaries of those claims through the liberal application of the
doctrine of equivalents.

34. Ravi Kamal Bali Vs Kala Tech And Ors., 2008 (110) Bom L.R. 2167

168
Facts:

The Plaintiff has sought an injunction restraining the Defendants from making
of, using, selling or distributing tamper proof locks/seals that fall within the
scope of the claims of the Plaintiff's patented invention titled "An improved
tamper proof seal for directly locking the container" under the trade mark
`TechLock' so as to infringe the same by dealing with similar products bearing
the name "SEAL TECH" which had constructional and functional features
similar to the Plaintiff's patented inventions.

Issue:

Whether there is infringement of patents applying the doctrine of equivalence


by which a device is set to infringe a claim if it "performs substantially the
same function in substantially the same way to obtain the same result” and
rule of pith and marrow of the invention. 

Judgment:

The Bombay High Court applied Doctrine of Equivalents and ruled in favour of
plaintiff ont he basis of same constructional and functional aspect of the two products.

Propositions:

 The marginal difference in the quality of steal does not account for any
difference in the invention if the usage or the purpose of the material produced
by the Plaintiff and Defendant are the same.
 The requirement of some amount of compression by pinching the two sides of
the `V' shaped body between the thumb and the index figure, would make no
difference and it is a mere cosmetic or superficial difference which does not
have any bearing on the invention.
 The provisions of vanes and the operating system viz. compression and
decompression are essential ingredients in the invention. While the term
"vanes" is not used in the claim of Defendant’s patent application, it is clear
that the Defendant considered the provision of vanes and the mode of
operation thereof to be essential to the invention.
 The difference in the shape of the main body does not by any stretch of
imagination constitute the Defendants product as a separate or a new
invention. The rectangular box shape in the Defendant's product than `V'
shaped body in  Plaintiff's product has no functional novelty.
 The number of vanes make any difference to the question of infringement.
 The provision of an outwardly facing notch/flap window for additional safety
also makes no difference to the question of infringement. It does not change
the essential functional system.
 Section 54 permits the grant of a patent of addition only if the applicant
thereof applies or has applied for a patent of the main invention in respect
whereof the notification or improvement is claimed. A stranger to the main
invention cannot apply for a patent of addition in respect of any modification
or improvement of the main intention.

169
Burden of proof in suit for patent infringement
35. Medtronic, Inc. V. Mirowski Family Ventures, LLC
(Certiorari to the United States Court of Appeals for the Federal Circuit; Argued
November 5, 2013—Decided January 22, 2014)
Burden of proof in suit for patent infringement by a patentee against the
licensee- A patentee ordinarily bears the burden of proving infringement.
Whether the burden of proof shifts when the patentee is a defendant in a
declaratory judgment action, and the plaintiff (the potential infringer-licensee)
seeks a judgment that he does not infringe the patent? Held that, when a licensee
seeks a declaratory judgment against a patentee to establish that there is no
infringement, the burden of proving infringement remains with the patentee.
Facts of the case:
Medtronic was sub-licensee of the defendant (patent holder). The plaintiff-
licensee sought a declaratory judgement in Federal District Court in Delaware
that he has not infringed the licensed patents of defendant. The District Court
believed that Mirowski, “as the party asserting infringement,” bore the burden
of proving infringement but the Court of Appeals for the Federal Circuit came
to the opposite conclusion. In that case, the Court of Appeals held, the party
seeking a declaratory judgment of non-infringement bears the burden of
persuasion(in this case Medtronic). So Medtronic sought a certiorari by
Supreme Court of United States wherein the SC was of the view, the burden of
persuasion is with the patentee, just as it would be had the patentee brought an
infringement suit.
Reasons for the conclusion:
i. burden of proving infringement generally rests upon the patentee
ii. The operation of the Declaratory Judgment Act was only procedural, and did
not change the substantive rights. And since burden of proof is a substantive
aspect of the claim it remains unchanged.
iii. One of the practical considerations to shift the burden depending upon the form
of the action could create post-litigation uncertainty about the scope of the
patent.

36. Virgin Atlantic Airways Ltd v Jet Airways (India) Ltd, [2013] UKSC 46
Facts:
1. Virgin is the owner of 908 patents which relates to seating systems for a
passenger aircraft in which the seats are arranged in what is called an
inward-facing herringbone configuration and in which each seat converts
into a lie-flat bed.
2. Virgin began infringement proceedings against Zodiac for its product
under the name “Solar Eclipse” (Contour Action).
3. The 908 patent also became the subject of opposition proceedings brought
in the EPO by Premium, Airbus Industrie and Cathay Pacific.

170
4. Zodiac denied infringement, and counterclaimed for revocation of the
Patent on the grounds, inter alia, that it was invalid in the light of prior art.
Those proceedings resulted in a hearing in front of the Court of Appeal,
which (i) decided that the Patent was valid, (ii) declared that Zodiac's
product infringed it, and (iii) ordered an assessment of damages– [2009]
EWCA Civ 1062 and 1513.
5. At the EPO the opposition proceedings came before the Technical Board
of Appeal (“TBA”). During the course of the hearing before the TBA,
Virgin abandoned the claims which Zodiac had been held to infringe, and
the Patent was amended accordingly.
Issue:
Whether Zodiac is entitled to contend upon the enquiry as to damages that
there have been no damages because the patent has been retrospectively
amended so as to remove the claims held to have been infringed?
As far as application of principle of res judicata was concerned the SC took
note of the fact the Court of Appeal decided, before the result of the opposition
proceedings in the EPO that in its unamended form the patent was valid and
infringed. It follows that Zodiac are estopped from asserting on the enquiry as
to damages that in its unamended form the patent was invalid favour of Virgin
issues essential to the existence of the cause of action for infringement of the
unamended patent, which are the basis of the claim for damages. However, the
point which Zodiac seeks to make on the enquiry is that the unamended patent
has been retrospectively amended. It no longer exists, and is deemed never to
have existed, in the form on which these issues were adjudicated by the Court
of Appeal. Zodiac's reliance on the retrospective amendment is a new point
which was not raised before. It could not have been raised before, because the
decision of the TBA retrospectively amending the patent was made after the
order giving effect to the judgment of the Court of Appeal. There are two
related reasons why Zodiac cannot be precluded from relying on the decision
of the TBA on the enquiry as to damages. One is that they are relying on the
more limited terms of a different patent which, by virtue of the decision of the
TBA, must at the time of the enquiry be treated as the only one that has ever
existed. The other is that Zodiac are not seeking to reopen the question of
validity determined by the Court of Appeal. The invalidity of the patent may be
the reason why the TBA amended the patent, but the defendant is relying on
the mere fact of amendment, not on the reasons why it happened. Principle of
res judicata not applicable in the instant case because Zodiac are not seeking to
relitigate or raise a point which was determined by the Court of Appeal in the
English proceedings. Zodiac are simply seeking to rely on a highly relevant
event which occurred after the determination of those proceedings, and which
self-evidently would have a very significant effect on the assessment.
SC also overruled decision of the Court of Appeal in Coflexip SA v Stolt
Offshore MS Ltd (2004) and Unilin Beheer BV v Berry Floor NV [2007] FSR
635 which had held that a patentee whose patent has been held to be valid is

171
entitled to claim damages for its infringement without regard to a subsequent
revocation of the patent. Held that “Where judgment is given in an English
court that a patent (whether English or European) is valid and infringed and the
patent is subsequently retrospectively revoked or amended (whether in England
or at the EPO), the defendant is entitled to rely on the revocation or amendment
on the enquiry as to damages.

37. FDC Limited & Ors v. Sanjeev Khandelwal & Ors, 2007 (35) PTC 436.


(Mad). 

As regards the burden of proof, a finding as to whether infringement has


occurred or not in a case is a question of fact and the onus is on the party
alleging infringement to adduce evidence of infringement. A mere allegation
that infringement has occurred, without any cogent evidence or pleading in
support thereof is not sufficient in law for a party to secure an injunction
against another. It is not enough for a patent holder to merely allege that there
has been infringement of his patent without any evidence of the said claim
especially when the subject mater of the alleged infringement is a process
which by its nature is invisible and hence cannot be compared side by side
during juridical scrutiny to arrive at a conclusion as to the infringement. Even
as per Section 104 A of the Patents Act, in any suit for infringement of a
patent, where the subject matter of patent is a process for obtaining a product,
the court may direct the defendant to prove that the process used by him to
obtain the product, identical to the product of the patented process, is different
from the patented process, only if (a) the subject matter of the patent is a
process for obtaining a new product; or (b) there is a substantial likelihood that
he identical product is made by the process, and the patentee or a person
deriving title or interest in the patent from him, has been unable through
reasonable efforts to determine the process actually used; provided that the
patentee or a person deriving title or interest in the patent from him firs proves
that the product s identical to the product directly obtained by the patented
process. In considering whether a party has discharged the burden imposed
upon him as above, the court shall not require him to disclose any
manufacturing or commercial secrets, if it appears to the court that it would be
unreasonable to do so.

38. Sandeep Jaidka vs Mukesh Mittal & Anr. DLHC CS(OS) No.1900/2010

Facts:

The invention of the plaintiff is a combination of a hardware and software,


which together are designed to control and synchronise the movie played for a
multi-sensory show through '4D' effects. It is the case of the plaintiff that the
defendants are wilfully infringing that said patent i.e. by introducing in their

172
4D theatre (called "funvilla" ) a device/system which
produces aromatic/heating, cooling and other effects corresponding to the
scene appearing in a movie, thus infringing plaintiff's patent and as a result of
these acts of wilful infringement, the plaintiff is suffering grave, irreparable
loss and damage.

Judgment:

The Delhi High Court disposed the application observing that no prima facie
case is made out by the plaintiff to grant injunction.

Issue:

Whether the theatre run by defendant No.1 employs the technology covered by
the plaintiff's patent No. 231895 and infringes plaintiff's patent?

Proposition:

 The Supreme Court in the case of Biswanath Prasad Radheyshyam v


Hindustan Metal Industries; PTC (Suppl) (1)731(SC) at Page 740 (para 32)
has observed that the grant and sealing of the patent, or the decision rendered
by the Controller in case of opposition, does not guarantee the validity of the
patent, which can be challenged before the High Court on various grounds in
the revocation or infringement proceedings.
 The Supreme Court further observed "It is important to bear in mind that in
order to be patentable an improvement on something known before or a
combination of different matters already known, should be something more
than a mere workshop improvement; and must independently satisfy the test of
invention or an 'inventive step'. To be patentable the improvement or the
combination must produce a new result, or a new article or a better or cheaper
article than before. 
 The "Gillette defence" arising out of the words of Lord Moulton in Gillette
Safety Razor Co. Vs. Anglo-American Trading Co. where he said : “The
defence that 'the alleged infringement was not novel at the date of the
plaintiff's letters patent', is a good defence in law, and it would sometimes
could and would put forth his case in this form, and thus spare himself the
trouble of demonstration on which horn of the well-known dilemma the
plaintiff had impaled himself, invalidity or non- infringement.”
 Kerr on Law and Practice of Injunction, 6th Edition on page 320 discusses
some principles which may act as guiding factors for the grant of injunction in
patent cases. The said factors are stated as follows:-
“If one clear instance of infringement or a wrong prima facie case
of infringement is made out and the plaintiff has not been guilty of
laches, the court will generally grant an interlocutory injunction in
following cases: (1) when the validity of the patent has already been

173
established in a previous action, (2) when the patent is of old standing
and the enjoyment under it has been uninterrupted (3) when the
validity of the patent is not in issue and notwithstanding that the
defendant offers to keep an account.”
 The plaintiff has not even filed any affidavit of any scientific/technical expert
regarding infringement.

 The same effects have been achieved world over by different ways, and
specially in Hollywood/Universal studios, since 1984 and the first 4D movie-
"The Sensorium" which contained multi sensory effects of wind, smell etc.
was produced in 1984.

 There has been complete non-use of the patented invention by the plaintiff 
and the Division Bench of this Court in Franz Xaver Huemer v. New Yash
Engineers, AIR1997 Delhi held that "a Plaintiff who has registered patent in
India but has not used them in India can not in equity seek temporary
injunction against the respondent".

39. Ram Kamal Bali v. Kala Tech, 2008 (110) Bom LR 216.

The instant case is perhaps the first case in India where the doctrine of
equivalents was applied in case non-literal infringement.

The plaintiff, in the instant case, was holding patents for “An improved tamper
proof seal for directly locking the container”, and a patent of addition for “An
improved tamper proof seal for directly locking the container having a lock
ring”. The defendants were manufacturing products with similar constructional
and functional features under the name ‘SEAL TECH’. The plaintiff contended
that the differences between the plaintiff’s products and the defendants’
products were minor, cosmetic and inconsequential, as the defendants’
products could not be designed independently of the plaintiff’s inventions. The
plaintiff urged the Court to apply the doctrine of equivalence and the tripartite
test in the given case to assess whether the defendants’ products “perform
substantially the same function in substantially the same way to obtain the
same result”. He further argued that the test was whether the “defendants’
product appears to have taken the essence or what is sometimes called the pith
and marrow” of the invention and submitted that even while deciding cases of
infringement under the Indian Patent Act, 1970, the Courts have applied the
doctrine of equivalence and the Court must determine and distinguish the
essential and non-essential elements of the product. The plaintiff emphasized
on the essence of the invention.

While the defendants denied the allegations of infringement, they did not
dispute the submissions of the plaintiff on the applicability of the doctrine of

174
equivalent. The defendants in fact submitted that even though their product
performed the same function and produced the same result, it did not however
perform the function in the same way as the plaintiff’s device. One of the
defendants (defendant no. 3) was a former employee of the plaintiff, having
unrestricted access to the technical and other conventional information relating
to the plaintiff’s business, who had joined the defendants (defendant no. 1
(Kala Tech) and 2 (sole proprietor of the Kala Tech)) to assist them in the
business. The plaintiff alleged that the defendant no. 3 had conspired with the
defendant no. 1 in the infringement of the plaintiff’s patents. The plaintiff
sought damages and an injunction restraining the defendants from making of,
using, selling or distributing such locks/seals falling within the scope of the
claims of the plaintiff’s patents. The defendants did not challenge the validity
of the plaintiff’s patents but submitted that the plaintiff was not entitled to the
injunction on the ground of delay and his conduct.

The High Court examined that stated object and similarities between the
plaintiff’s and defendant’s products in the light of the purpose, material of
construction, principle of working and construction features of the products.
The Court found that the two products functioned on the same principle of
compression and decompression. The defendants argued that the mode of
compression and decompression of the two products was different but the
Court rejected this argument as the difference was found to be minor and
cosmetic

EXCEPTION TO INFRINGEMENT OR DEFENSES TO INFRINGEMENT


40. Roche Products, Inc., v. Bolar Pharmaceuticals Co. (Fed. Cir. 1984)
In this case the Plaintiff-appellant Roche Products, Inc. (Roche), a large
research-oriented pharmaceutical company, wanted the United States district
court to enjoin Bolar Pharmaceutical Co., Inc. (Bolar), a manufacturer of
generic drugs, from taking, during the life of a patent, the statutory and
regulatory steps necessary to market, after the patent expired, a drug equivalent
to a patented brand name drug. Roche argued that the use of a patented drug for
federally mandated premarketing tests is a use in violation of the patent laws.
Basically Roche held patent rights over flurazepam hydrochloride
(flurazepam hcl), the active ingredient in Roche's successful brand name
prescription sleeping pill ―Dalmane.‖ The patent over the same was to expire
on January 17, 1984. In early 1983, Bolar became interested in marketing, after
the '053 patent expired, a generic drug equivalent to Dalmane. Because a
generic drug's commercial success is related to how quickly it is brought on the
market after a patent expires, and because approval for an equivalent of an
established drug can take more than 2 years, Bolar, not waiting for the '053
patent to expire, immediately began its effort to obtain federal approval to

175
market its generic version of Dalmane. In mid-1983, Bolar obtained from a
foreign manufacturer 5 kilograms of flurazepam hcl to form into ―dosage form
capsules, to obtain stability data, dissolution rates, bioequivalency studies, and
blood serum studies‖ necessary for a New Drug Application to the United States
Food and Drug Administration (FDA). As stated above, Roche argued that the
use of a patented drug for federally mandated premarketing tests is a use in
violation of the patent laws.
The District Court held that, ―Bolar's use of the patented compound for
federally mandated testing was not infringement of the patent in suit because
Bolar's use was de minimis and experimental.‖
On appeal before the CAFC, Bolar argued that its intended use of flurazepam
hcl is excepted from the use prohibition. It claims two grounds for exception:
the first ground is based on a liberal interpretation of the traditional
experimental use exception; the second ground is that public policy favors
generic drugs and thus mandates the creation of a new exception in order to
allow FDA required drug testing.
Tracing the history of the experimental use doctrine, the Court of Appeals
stated that, ―The aim to impose liability for infringement is to prevent
promotion of interest by the wrongful employment of the invention which is
hostile to the interest of the patentee. The interest of the patentee is represented
by the emoluments which he does or might receive from the practice of the
invention by himself or others. These, though not always taking the shape of
money, are of a pecuniary character, and their value is capable of estimation like
other property. Hence, acts of infringement must attack the right of the patentee
to these emoluments, and either turn them aside into other channels or prevent
them from accruing in favor of any one. An unauthorized sale of the invention is
always such an act. But the manufacture or the use of the invention may be
intended only for other purposes, and produce no pecuniary result.
Thus, where an invention is made or used as an experiment, whether for the
gratification of scientific tastes, or for curiosity, or for amusement, the interests
of the patentee are not antagonized, the sole effect being of an intellectual
character in the promotion of the employer's knowledge or the relaxation
afforded to his mind. But if the products of the experiment are sold, or used for
the convenience of the experimentor, or if the experiments are conducted with a
view to the adaptation of the invention to the experimentor's business, the acts
of making or of use are violations of the rights of the inventor and infringements
of his patent.
Thus, applying this reasoning the CAFC refused to interpret experimental
use in a broad manner and held that experiments [which] are in keeping with the
legitimate business of the [alleged infringer] are infringements for which
“[e]xperimental use is not a defense. The CAFC also held that “we cannot
construe the experimental use rule so broadly as to allow a violation of the
patent laws in the guise of ‘scientific inquiry,’ when that inquiry has definite,
cognizable, and not insubstantial commercial purposes.‖ Thus, the court

176
concluded that, ―Bolar may intend to perform “experiments,” but unlicensed
experiments conducted with a view to the adaption of the patented invention to
the experimentor's business is a violation of the rights of the patentee to exclude
others from using his patented invention. It is obvious here that it is a misnomer
to call the intended use de minimis. It is no trifle in its economic effect on the
parties even if the quantity used is small.”
41. Eli Lilly & Co. v. Medtronic Inc 496 U.S. 661 (1990)
Post the Roche decision the Congress enacted the Hatch Waxman Act
(1984), under Section 271(e) (1), which exempted from infringement ―uses
reasonably related to the development and submission of information under a
Federal Law which regulates manufacture, sale or use of drugs.‖
In this case, the District Court interpreted Section 271(e)(1) of the Hatch
Waxman Act. Lily sued Medtronic for infringement of its two patents covering
implantable cardiac defibrillators. Medtronic defended its activities based on the
experimental use shield of Section 271(e)(1). The court sided with Lily stating
that Section 271(e)(1) did not apply to medical devices. Relying on the
combination of legislative intent and Roche, the CAFC reversed the court‘s
decision. It noted that Congress explicitly stated that Section 271(e)(1) would
have the net effect of reversing the holding in Roche. Thus, under the decision
the court held that by way of the enactment, Congress meant to allow testing
and investigation of a patented invention if strictly related to obtaining approval
for a substitute, regardless of the product involved. The test being whether one
would reasonably believe that the product of research would tend to FDA
approval/acceptance, if yes then it is exempted. Hence, medical devices fell
within the purview of Section 271(e)(1).

43. Merck KGaA v. Integra Lifesciences Inc., Ltd., 125 Sct 2372.
The Respondents in the Supreme Court case, Integra LifeSciences I, Ltd.,
and the Burnham Institute (collectively Integra), own five patents that cover
peptides containing the sequence known as RGD peptides.
The petitioner, Merck KGaA, sponsored research at the Scripps Research
Institute conducted by Dr. David Cheresh in order to discover drugs that inhibit
angiogenesis for use as anti-cancer drugs. Dr. Cheresh conducted experiments
and narrowed down to one of the peptides as the most promising peptides for
testing on humans. The National Cancer Institute filed an IND with the FDA for
the selected peptide. The IND is a document which must describe ―preclinical
tests (including tests on animals) of [the] drug adequate to justify the proposed
clinical testing.
Integra filed a patent infringement suit against Merck, Scripps, and Dr.
Cheresh. Integra alleged that Merck infringed, and induced others to infringe,
Integra‘s patents by supplying the RGD peptides to Scripps, and that Dr.

177
Cheresh and Scripps infringed the patents by using the RGD peptides in the
angiogenesis experiments.
Merck argued that the safe harbour under the Hatch Waxman Act protected
it. The District Court interpreted the safe harbour principle as follows: ―To
prevail on this defence, [Merck] must prove by preponderance of the evidence
that it would be objectively reasonable for a party in Merck‘s and Scripp‘s
situation to believe that there was a decent prospect that the accused activities
would contribute, relatively directly, to the generation of the kinds of
information that are likely to be relevant in the processes by which the FDA
would decide whether to approve the product in question. Despite the broad
worded exception, the Jury decided against Merck and found it guilty of patent
infringement.
On appeal, the Federal Circuit framed the following issue, whether s.271 safe
harbour reaches back down the chain of experimentation to embrace
development and identification of new drugs, that will, in turn be subject of
FDA approval. The Federal Circuit gave a narrow interpretation to the safe
harbour and held that the safe harbour provision ―does not, for instance,
expand the phrase ‘reasonably related’ to embrace the development of new
drugs because those new drugs will also need FDA approval. Thus, the safe
harbour does not globally embrace all experimental activity that at some point,
however, attenuated, may lead to an FDA approval process. Thus, only those
activities are exempted under the safe harbours which are reasonably related to
acquiring FDA approval of a drug already in the market.
It is pertinent to note that the decision of the Federal Circuit gains
importance especially with reference to use of patented research tools, with the
narrow interpretation of the safe harbor, use of such tools would fall outside the
exception.
The matter went up to the Supreme Court. Justice Scalia, delivering the
opinion for the court held that the issue was as follows: whether use of patented
inventions in preclinical research, the results of which are not ultimately
included in the submission to the FDA, are exempted from infringement by the
safe harbor provisions. Justice Scalia disagreed with the narrow interpretation
given by the Federal Court and followed the broad and loose test laid down by
the District Court. According to the Supreme Court, the safe harbor covers a
broad range of pre-clinical as well as clinical studies.
Stating the parameters of the test broadly, the Apex Court held that, “at least
where a drug maker has a reasonable basis for believing that a patented
compound may work, through a particular biological process. To produce a
particular physiological effect, and uses the compound in research that, if
successful would be appropriate to include in a submission to the FDA, that use
is “reasonably related” to the mandate under Section 271(e)(1)”.
On the basis of this standard, the Apex Court differentiated pre-clinical trials
from basic scientific research and held that basic scientific research on a
particular compound, performed without the intent to develop a particular drug

178
or a reasonable belief that the compound will cause the sort of physiological
effect the researcher intends to induce, is surely not “reasonably related” to the
mandate prescribed under Section 271(e)(1)”

44. Bayer Corporation Vs. Natco Pharma Limited, [November 2014]the High
Court of Delhi examined the scope of India’s Bolar exemption. [Excerpt taken
from:- https://iiprd.wordpress.com/tag/bayer-corporation-vs-natco-pharma-limited/ ]

Facts:

Petitioner (Bayer Corporation) was granted a patent (IN215758) for a


pharmaceutical product titled “Nexavar” (compound being Sorafenib
Tosylate) which is used for the treatment of patients with advanced stages of
kidney and liver cancer. Natco (respondent no. 5) applied for and was granted
a “compulsory license” to manufacture pharmaceutical products which were
covered under the Bayer’s patent on the ground that reasonable requirement of
the public with respect to the aforesaid product was not met and it was not
made available at reasonable and affordable price. Specifically, the
“compulsory license” was “solely for the purpose of making, using, offering
for sale and selling the drug covered by the patent for the purpose of treating
HCC and RCC in humans within the territory of India”. Subsequently, Natco
started manufacturing the product under the brand name ‘Sorafenat’ but was
not permitted to export the same.

Bayer allegedly received information that Natco was exporting the API,
Sorafenib Tosylate outside India in violation of the terms of the compulsory
license. Consequently, Bayer filed a writ petition with the Court requesting
that the product covered under the compulsory license be confiscated and any
consignment for export be seized. On March 26, 2014, the Court passed an
interim order directing Natco to ensure that no consignment containing
Sorafenib Tosylate covered by the compulsory license was exported.

In the mean time, an application was filed by Natco before the court inter alia
praying for permission to export 1 kg of Sorafenib Tosylate (API) to a Chinese
pharmaceutical company for the purpose of conducting development/clinical
studies and trials. Natco produced a certificate from the Chinese company
stating that the company required 1 kg of Sorafenib for the purpose of
“formulation R&D purpose and the same was not intended for any commercial
purpose”.

Natco argued that the 1 kg of Sorafenib was required for conducting


regulatory studies in China and despite the compulsory license, export of the
API was exempt by virtue of Section 107A of the Patents Act, 1970. Bayer
argued that the regulatory approvals in China were being sought by the
Chinese Pharma company and not by Natco, and hence Section 107A would
not apply. Bayer further argued that exportation of Sorafenib was not
permitted by the terms of the compulsory license and would not fall within the
scope of Section 107A. Bayer stressed that the use of Sorafenib by a third
party for development purposes did not entitle Natco to infringe Bayer’s
patent. Additionally, Bayer argued that because Natco was not conducting any

179
development studies, exportation of Sorafenib to China could not be construed
to be solely for uses related to the development and submission of information
required by any regulatory authority. Finally, Bayer noted that Section 107A
did not permit export of Bayer’s product because the word “export” was
missing from the section 107A.

Issue:

Specifically, the issue to be decided in this case was whether the exportation
of an active pharmaceutical ingredient (API) constituted a defense to
infringement under Section 107A of the Patents Act.           

Judgment:

After a review of the facts, the Court accepted that the product Natco sought to
export was not for commercial purposes since the amount was only sufficient
to make 1000 to 2000 tablets (which was approximately the single trial batch
size required by the Chinese Regulatory Authorities). Thus, according to the
Court, the only question that had to be addressed was whether Section 107A
covered export of a patented product for use by an overseas importer to
conduct studies and generate data for the purpose of seeking regulatory
approval in that country.

Bayer relied heavily on the history of Bolar exemption in US (which allows


sale only in the United States) and the Polish decision ‘Polpharma’ to contend
that the applicability of Section 107A of the Act must be restricted to only
self-use.

The Court rejecting the reliance placed by Bayer on ‘Polpharma’, held that a
sale which is related to submission of information as required by any law in
India or outside India would be permissible by virtue of Section 107A. The
Court further stated that the exclusion to a patentee’s right as provided under
Section 107A is wider than the exceptions provided by the laws of the U.S.

Specifically, the Court stated:

“India is one of the largest producers of generic versions of drugs around the
world. Given the economic realities of our country, providing cheaper
medicines is a necessity. The parliament in its wisdom has, thus couched the
exclusion to a patent, as provided under Section 107A, in wide terms. The
sweep of the plain language of Section 107A, thus, cannot be restricted in the
manner as canvassed on behalf of Bayer.

Propositions:

 …Plainly, Section 107A of the Act takes within its fold any sale of a patented
invention which is required for development and submission of information
under any law in a country other than India that regulates the manufacture or
sale of any product. Indisputably, under the Chinese Law, submission of
studies and data related to bio-equivalence and bio-availability of API in a

180
generic version, is required as discussed earlier and the sale of 1 kg. of
Sorafenib to Chinese company can be reasonably stated to be related to the
studies that are required to be conducted by Chinese company for obtained the
regulatory approvals.
 …[t]he language of Section 107A of the Act is determinative of the question
whether export as sought for by Natco is permissible within the exemption of
Section 107A of the Act. The use of the expression ‘reasonably related to’ as
used in Section 107A of the Act would plainly mean a reasonably nexus.
Thus, the only question that needs to be answered is whether there is any
reasonable nexus between the sale of Sorafenib by Natco to Chinese company
and submission of information under the law in force in China. In my view,
the answer to this question is clearly in the affirmative.
 …It is also important to note that the language of Section 107A of the Act is
materially different from the law as applicable in U.S. Whilst, the US Law
restricts the safe harbour to a sale within United States and solely for purposes
related to information under a Federal Law, Section 107A of the Act is
circumscribed by no such conditions. Thus, a sale even outside India would
fall within the sweep of Section 107A, provided it is reasonably related to
development and submission of information as required under a law in force in
India or outside India.”
   The Court also rejected Bayer’s arguments that the language of Section 107A
excluded “exports” because this term was not specifically recited. However,
the Court stated:
 “I am not inclined to accept this contention for the reason that the expression
‘selling’ is wide enough to even include cross border sales (i.e. exports). If the
Parliament intended to restrict the exception to only sales within India, the
same would have been expressly stated as was done by the US Congress under
35 US Code 271(e)(1).”
  The court stated that the sweep of the plain language of Section 107A, thus,
cannot be restricted in the manner as canvassed on behalf of Bayer.
Eventually, the court allowed the application filed by Natco on the ground that
the sale of 1 kg of Sorafenib Tosylate to the Chinese pharmaceutical company,
in China could be reasonably stated to be related to the studies that are
required to be conducted for obtaining the regulatory approvals. The Delhi
High Court interpreted Section 107A expansively to conclude that Section
107A is applicable when a party exports a patented product to a third party
outside India as long as the purpose of export is the facilitation of research.

Doctrine of Patent exhaustion


45. Bowman v. Monsanto Co, US SC decided on 13 May, 2013
Facts: Monsanto invented a genetic modification that enables soybean plants
to survive exposure to glyphosate, the active ingredient in many herbicides.
Monsanto markets soybean seed containing this altered genetic material as
Roundup Ready seed. Farmers planting that seed can use a glyphosate based
herbicide to kill weeds without damaging their crops.
It sells the seeds subject to a licensing agreement that permits farmers to
plant the purchased seed in one, and only one, growing season. Growers may

181
consume or sell the resulting crops, but may not save any of the harvested soy-
beans for replanting.
Bowman purchased Roundup Ready soybean seed for his first crop of each
growing season from a company associated with Monsanto and followed the
terms of the licensing agreement. But to reduce costs for his riskier late-season
planting, Bowman went to a grain elevator; purchased “commodity soybeans”
intended for human or animal consumption; and planted them in his fields. This
was possible because soybean itself was a seed. He planted them; treated the
plants with glyphosate, killing all plants without the Roundup Ready trait;
harvested the resulting soybeans that contained that trait; and saved some of
these harvested seeds to use in his late-season planting the next season. After
discovering this practice, Monsanto sued Bowman for patent infringement.
Bowman raised the defense of patent exhaustion, which gives the purchaser of a
patented article, or any subsequent owner, the right to use or resell that article.
The District Court rejected that argument, and awarded damages to Monsanto of
$84,456. The Federal Circuit affirmed and reasoned that the “right to use” a
patented article following an authorized sale does not include the right to
construct an essentially new article on the template of the original, for the right
to make the article remains with the patentee. Accordingly, Bowman could not
“‘replicate’ Monsanto’s patented technology by planting it in the ground to
create newly infringing genetic material, seeds, and plants.”
SC granted certiorari and affirmed the decision of Federal Circuit. SC
explained that the doctrine of patent exhaustion limits a patentee’s right to
control what others can do with an article embodying or containing an invention
but the doctrine restricts a patentee’s rights only as to the “particular article”
sold; it leaves untouched the patentee’s ability to prevent a buyer from making
new copies of the patented item. The exhaustion doctrine does not extend to the
right to ‘make’ a new product. The exhaustion doctrine is limited to the “par-
ticular item” sold to avoid just such a mismatch between invention and reward.

46. F. Hoffmann-La Roche Ltd. And Anr. vs Cipla Limited IA 642/2008

Facts:

The Plaintiffs in this suit seek permanent injunction restraining infringement


of their patent rights in the drug Erlotinib (a major breakthrough and
innovation in the treatment of cancer), rendition of accounts, damages and
delivery up of the infringing goods by CIPLA by offering a generic version of
the patented drug.

Issue:

Whether the claimed drug significantly differ in efficacy compared to the prior
art  and hence patentable in view of S. 3(d) of Indian Patent Act, 1970?

Judgment:

182
The Delhi High Court dismissed the application holding that the invention
cannot be held non-patentable under Section 3(d) of the Patents Act, 1970.

Proposition:

 The amended explanation was considered by a judgment of the Division


Bench of the Madras High Court, in Novartis A.G. v Union of India (4) MLJ
1153. The court held that: “in sum and substance what the amended section
with the Explanation prescribes is the test to decide whether the discovery is
an invention or not is that the Patent applicant should show the discovery has
resulted in the enhancement of the known efficacy of that substance and if the
discovery is nothing other than the derivative of a known substance, then, it
must be shown that the properties in the derivatives differ significantly with
regard to efficacy.”

 The Parliament consciously enacted the standard of non- obviousness as a


condition for patentability; it also excluded some matter, i.e., derivatives of
substances which are known to exist, unless they differ in properties,
significantly, in the known efficacy. Thus, it has to be concluded that the test
of non-obviousness of an invention and discovery of existence of significant
enhancement in the known efficacy of a substance are pre-requisites of
patentability. In other words, even if non-obviousness of an invention in the
pharmaceutical or Chemical industry are established, the applicant should also
prove that if the invention claimed is the derivative of a known substance, it
does not fall within the excepted category, in the Explanation to Section 3(d)
as it comprehends a discovery of significant enhancement in known efficacy
of such known substance. 

 In the United Kingdom, the Court of Appeal in, Windsurfing International Inc.
v. Tabur Marine (GB) Ltd. [1985] RPC 59, required the following steps to be
taken into account while determining patentability: 1. Identifying the inventive
concept embodied in the patent; 2. Imputing to a normally skilled but
unimaginative addressee what was common general knowledge in the art at
the priority date; 3. Identifying the differences if any between the matter cited
and the alleged invention; and 4. Deciding whether those differences, viewed
without any knowledge of the alleged invention, constituted steps that would
have been obvious to the skilled man or whether they required any degree of
invention.

 The U.S. Supreme Court in Grahametal. v. John Deere Co. of Kansas City et
al. 383 U.S. 1 (1966) held that obviousness should be determined by looking
at 1.the scope and content of the prior art; 2.the level of ordinary skill in the
prior art; 3.the differences between the claimed invention and the prior art; and
4.objective evidence of non-obviousness. In addition, the Court outlined
factors that show ``objective evidence of non- obviousness'`. They are: 1.
commercial success; 2. long-felt but unsolved needs; and 3. failure of others.

New Product- Section 3(d)-Ever greening of Patents

183
47. Novartis AG. v. Union of India & Others (Decided By Supreme Court of
India On 01/04/13)
Facts: Jürg Zimmermann invented Imatinib which had anti-tumor properties.
Patent was sought for Imatinib Mesylate in beta crystalline form at the Chennai
Patent Office on July 17, 1998. In the application it claimed that the invented
product, the beta crystal form of Imatinib Mesylate, has (i) more beneficial flow
properties: (ii) better thermodynamic stability; and (iii) lower hygroscopicity
than the alpha crystal form of Imatinib Mesylate. It further claimed that the
aforesaid properties make the invented product new (and superior).
Meanwhile it was granted EMR but when the patent application was taken
out of mailbox for consideration, five pre grant oppositions had been filed
against it. And it was in response to the pre-grant oppositions that the appellant
had filed the affidavits on the issue of bioavailability of Imatinib Mesylate in
beta crystalline form stating that the beta crystal form of Imatinib Mesylate has
much higher bioavailability as compared to Imatinib in free base form (No such
claim was however made in original patent application).
The Assistant Controller held that the invention claimed by the appellant
was anticipated by prior publication, i.e., the Zimmermann patent; that the
invention claimed by the appellant was obvious to a person skilled in the art in
view of the disclosure provided in the Zimmermann patent specifications; and
further that the patentability of the alleged invention was disallowed by
section 3(d) of the Act. Appellant appealed against the order and also
challenged the constitutional validity of S 3(d) before Madras High Court on
the grounds that it not only violates Article 14 of the Constitution of India but
is also not in compliance with “TRIPS”. The writ was dismissed and vires of
S 3(d) were upheld. IPAB reversed the findings of the Assistant Controller on
the issues of anticipation and obviousness. It held that the appellant’s
invention satisfied the tests of novelty and non-obviousness The IPAB,
however, held that the patentability of the subject product was hit by section
3(d). Against the order of the IPAB the appellant came directly to Supreme
Court in a petition under Article 136 of the Constitution.
SC held that:
1. Imatinib Mesylate (marketed as Gleevec) is not a new product.
To come to this conclusion SC took note of certain publications and the fact
that since the grant of the Zimmermann patent, the appellant has maintained
that Gleevec (that is, Imatinib Mesylate) is part of the Zimmermann patent. It
obtained drug approval for Gleevec on that basis. It claimed extension of the
term of the Zimmermann patent for the period of regulatory review for
Gleevec, and it successfully stopped NATCO Pharma Ltd. from marketing its
drug in the UK on the basis of the Zimmermann patent. Further it was noticed
that after the Zimmermann patent, the appellant applied for, and in several
cases obtained, patent in the US not only for the beta and alpha crystalline
forms of Imatinib Mesylate, but also for Imatinib in a number of different
forms. The appellant, however, never asked for any patent for Imatinib

184
Mesylate in non-crystalline form, for the simple reason that it had always
maintained that Imatinib Mesylate is fully a part of the Zimmermann patent
and does not call for any separate patent.
Not only is Imatinib Mesylate known as a substance in the Zimmermann
patent, but its pharmacological properties are also known in the Zimmermann
patent and in the article published in the Cancer Research journal. The
consequential finding, therefore, is that Imatinib Mesylate does not qualify the
test of “invention” as laid down in section 2(1)(j) and section 2(1)(ja) of the
Patents Act, 1970.
2. Beta crystal form of Imatinib Mesylate is hit by Sec 3(d) of the Patents
Act, 1970.
With respect to efficacy argument the court explained that the test of
efficacy would depend upon the function, utility or the purpose of the product
under consideration. Therefore, in the case of a medicine that claims to cure a
disease, the test of efficacy can only be “therapeutic efficacy. Therefore not
all advantageous or beneficial properties are relevant, but only such properties
that directly relate to efficacy, which in case of medicine is its therapeutic
efficacy. in this case the physico-chemical properties of beta crystalline form
of Imatinib Mesylate, namely (i) more beneficial flow properties, (ii) better
thermodynamic stability, and (iii) lower hygroscopicity, may be otherwise
beneficial but these properties cannot even be taken into account for the
purpose of the test of section 3(d) of the Act, since these properties have
nothing to do with therapeutic efficacy.
The finding to this effect was also based on the fact that the patent
application submitted by the appellant contained a clear and unambiguous
averment that all the therapeutic qualities of beta crystalline form of Imatinib
Mesylate are also possessed by Imatinib in free base. So there was no
question of enhanced efficacy of a known substance in new form.
As far as the contention of enhanced bioavailability was concerned the
court came to the conclusion that just increased bioavailability alone may not
necessarily lead to an enhancement of therapeutic efficacy. Whether or not an
increase in bioavailability leads to an enhancement of therapeutic efficacy in
any given case must be specifically claimed and established by research data.
In this case there was no material on record to show enhancement in
bioavailability also led to enhancement of therapeutic effect.
48. Apple Inc. v. Samsung Electronics Co., 678 F.3d 1314 (Fed. Cir. 2012)
It was an appeal from the denial of a preliminary injunction to block the
importation into and sale within the United States of the accused Samsung
devices. However CAFC took note of the established law that “A plaintiff
seeking a preliminary injunction must establish that he is likely to succeed on
the merits, that he is likely to suffer irreparable harm in the absence of
preliminary relief, that the balance of equities tips in his favor, and that an
injunction is in the public interest.” The decision to grant or deny a
preliminary injunction lies within the sound discretion of the district court, and

185
it will not reverse its judgment absent an abuse of that discretion. After
examining each of the alleged infringements the court affirmed the denial of
preliminary injunction with respect to 3 patents and with respect to the fourth
one vacated the order denying an injunction and remanded the case to the
district court for further proceedings on that portion of Apple’s motion for
preliminary relief.
49. Akamai Technologies Inc. v. Limelight Networks Inc.
(US Court of Appeals for the Federal Circuit) Decided on August 31, 2012
Facts: Akamai Technologies, Inc, owns a patent that covers a method for
efficient delivery of web content. The claimed method consists of placing some
of a content provider’s content elements on a set of replicated servers and
modifying the content provider’s web page to instruct web browsers to retrieve
that content from those servers. Akamai filed a complaint against Limelight
Networks, Inc, alleging infringement of the patent. In its complaint, Akamai
alleged both direct and induced infringement. Limelight maintains a network of
servers and, as in the patented method, it allows for efficient content delivery by
placing some content elements on its servers. Limelight, however, does not
modify the content providers’ web pages itself. Instead, Limelight instructs its
customers on the steps needed to do that modification.
The district court held Limelight not to infringe the patents because
Limelight’s customers (and not Limelight itself) performed one of the steps of
the claimed method. It was guided to this conclusion by BMC case which had
laid down that in order for a party to be liable for induced infringement, some
other single entity must be liable for direct infringement.
Question for consideration:
In case of method patent, whether a defendant may be held liable for induced
infringement if the defendant has performed some of the steps of a claimed
method and has induced other parties to commit the remaining steps and when
no single “induced” entity commits all of the infringing acts or steps but where
the infringing conduct is split among more than one other entity.
Held:
The Federal Court reviewed the decision of the Court in BMC Resources,
Inc. v. Paymentech, L.P., 498 F.3d 1373 (Fed. Cir. 2007) wherein it was held
that in order to support a finding of induced infringement1, not only must the
inducement give rise to direct infringement, but in addition the direct
infringement2 must be committed by a single actor.
The court overruled the decision in BMC case in as much as it required that
only a single actor must commit direct infringement. It reasoned that if a party
has knowingly induced others to commit the acts necessary to infringe the

1
A person is liable under 35 U.S.C. S. 271(b) for indirect infringement when a single actor
induces another actor to commit all the elements of infringement

2
A person is liable under 35 U.S.C. S. 271(a) for direct infringement when a single actor
commits all the elements of infringement.

186
patent and those others commit those acts, there is no reason to immunize the
inducer from liability for indirect infringement simply because the parties have
structured their conduct so that no single defendant has committed all the acts
necessary to give rise to liability for direct infringement. The court concluded
that Limelight would be liable for inducing infringement if the patentee could
show that (1) Limelight knew of Akamai’s patent, (2) it performed all but one
of the steps of the method claimed in the patent, (3) it induced the content
providers to perform the final step of the claimed method, and (4) the content
providers in fact performed that final step. So the case was remanded to the
district court.
However, the SC granted a certiorari and arguments have been heard with
respect to indirect infringement and a decision is awaited.

Other Important Case Laws Related to Patent Infringement In Indian


Context

50. Bishwanath Prasad Radhey Shyam vs Hindustan Metal Industries AIR


1982, SC 1444

Facts:

M/s. Hindustan Metal Industries, respondent herein, (hereinafter called the


plaintiff) is a registered partnership firm carrying on the business of
manufacturing brass and German silver utensils has prayed for a permanent
injunction restraining Biswanath Prasad Radhey Shyam, appellant herein,
(hereinafter called the defendant) from adopting, imitating, employing or in
any manner infringing the device of the plaintiff's patent namely, that of a
lathe (headstock, adapter and tailstock).

Issue:

Whether the "manner of manufacture" patented, was publicly known, used and
practised in the country before or at the date of the patent ?

Judgement:

The Supreme Court of India allowed the appeal ruling that the invention got
patented by M/s. Hindustan Metal Industries, respondent herein, was neither a
manner of new manufacture, nor a distinctive improvement on the old
contrivance involving any novelty or inventive step having regard to what was
already known and practised in the country for a long time before 1951.

Propositions:

 Encyclopaedia Britannica, Vol. 17, page 453: “A patent can be granted only
for 'manner of new manufacture' and although an invention may be 'new' and
relate to a 'manner of manufacture' it is not necessarily a 'manner of new
manufacture'-it may be only a normal development of an existing

187
manufacture. It is a necessary qualification of a craftsman that he should have
the knowledge and ability to vary his methods to meet the task before him-a
tailor must cut his cloth to suit the fashion of the day-and any monopoly that
would interfere with the craftsman's use of his skill and knowledge would be
intolerable.”
 The 'obviousness' has to be strictly and objectively judged. For this
determination several forms of the question have been suggested. The one
suggested by Salmond L. J. in Rado v. John Tye & Son Ltd. is apposite. It is:
"Whether the alleged discovery lies so much out of the Track of what was
known before as not naturally to suggest itself to a person thinking on the
subject, it must not be the obvious or natural suggestion of what was
previously known." Another test of whether a document is a publication which
would negative existence of novelty or an "inventive step" is suggested, as
under: "Was it for practical purposes obvious to a skilled worker, in the field
concerned, in the state of knowledge existing at the date of the patent to be
found in the literature then available to him, that he would or should make the
invention the subject of the claim concerned ?" 
 As pointed out in Arnold v. Bradbury the proper way to construe a
specification is not to read the claims first and then see what the full
description of the invention is, but first to read the description of the invention,
in order that the mind may be prepared for what it is, that the invention is to be
claimed, for the patentee cannot claim more than he desires to patent. In
Parkinson v. Simon Lord Esher M. R. enunciated that as far as possible the
claims must be so construed as to give an effective meaning to each of them,
but the specification and the claims must be looked at and construed together.
 Affirming the judgment of trial bench, the court restated “There is hardly any
difference between fixing the headstock and tailstock to a common base (as
the case in the machine Ex. XVI produced by Bhagwati Prasad) or fixing the
tailstock in a bracket which is connected with the framework on which the
headstock is fixed. Whether we consider Ex. CC as a whole or look at the
invention in its separate parts, we do not find any novelty in the alleged
invention.”
 "There is hardly any difference between fixing the headstock and tailstock to a
common base (as the case in the machine Ex. XVI produced by Bhagwati
Prasad) or fixing the tailstock in a bracket which is connected with the
framework on which the headstock is fixed. Whether we consider Ex. CC as a
whole or look at the invention in its separate parts, we do not find any novelty
in the alleged invention."

51. Lallubhai Chakubhai Jariwala vs Chimanlal Chunilal And Co. AIR 1936
Bom 99

Facts:

Plaintiff is the holder of a patent for an improved process of treating dried


fruits, the process being particularly applicable to the treatment of dried shell
almonds and betel-nuts. The plaintiff's case is that defendants have been using
his process, or substantially the same process, for the treatment of betel-nuts,
and have been selling betel-nuts in the market treated by that process. 

188
Issue:

Whether the plaintiff claims his patent for a new combination made up of its
component parts, sometimes called the subordinate integers, or whether the
first claim in the specification is a separate invention by itself which has also
been patented and therefore entitled to protection? Whether the plaintiff's
invention has been anticipated by prior public user?

Judgment:

The Bombay High Court dismissed the appeal holding that since the use of
pressure by the defendants in their process has not been satisfactorily proved,
and hence there is therefore no infringement of the combination which is
validly patented product of plaintiff.

Proposition:

 It has been pointed out by Lord Esher, M.R., in Parkinson v. Simon (1894) 11
R.P.C 493 that as far as possible the claims must be so construed as to give an
effective meaning to each of them, but the specification and the claims must
be looked at and construed together in order to see whether there is a real and
substantial difference between the-claims, or whether one is practically a
repetition of the other. 
 The court reaffirmed the judgment of the Appeal Court that the plaintiff's
ingenuity and skill consisted in combining substances more or less previously
known in a particular manner and in a particular sequence so as to prepare and
produce an article which had not been produced before, and in overcoming the
difficulties which existed even after a crude application of some of those
substances previously. 
 There must, be an absence of prior publication or of prior public user ; and if
the user is secret or experimental, the profits made by the inventor from his
invention must not be excessive.
 There is not, in my opinion, sufficient and cogent evidence to establish that
before the application plaintiff had made his process ^ publicly known, nor
any reason for doubting the plaintiff's word that secrecy was observed. Nor
has it been shown that betel-nuts were treated "by other merchants according
to the plaintiff's process before the date of the application.
 It may not be necessary that members of the public should have actually read
it. it is enough if the publication is accessible to the public without much
trouble, e.g., if the document is to be found in the library of the Government
Patent Office in Calcutta, or on the shelves of a public library in any known
place in India, or of a library appertaining to an educational or scientific
institution and easily accessible. With regard to the sufficiency of the
knowledge, the earlier publication must give the requisite knowledge clearly,
and it is not enough that it merely gives the means pi attaining such
knowledge. It must give sufficient information to a workman skilled r in the
particular art or craft in order to enable him to carry out the invention.
 The pith and essence of his process is the combination of the use of bleaching
powder and sulphur dioxide under pressure, and in order to show the

189
infringement he must prove-that the defendants have pirated the substance of
his invention. 
 There is no evidence in this case that anyone communicated the plaintiff's
process to the defendants, nor has anyone been called on his behalf to show
that the defendants' process is in all its essential features identical with that of
the plaintiff.
52. Bajaj Auto Ltd., vs TVS Motor Company Ltd. 2008) ILLJ 726 Mad,
MIPR 2008 (1) 217, 2008 (36) PTC 417 Mad

Facts:

There are two suits filed by Bajaj Auto Limited and TVS Motor Company
Limited against each other for the relief of permanent injunction in respect of
the plaintiff's patent (in first suit) and/or from using the technology/invention
described in the said patent and/or manufacturing, marketing, selling, offering
for sale or exporting 2/3 wheelers, including the proposed 125-CC FLAME
motorcycle containing an internal combustion engine or any internal
combustion engine or product and (in second suit) for declaring that the threats
held out by the defendant in that the plaintiff is infringing the defendant's
patent are unjustified, for declaring that the plaintiff's product TVS Flame
which uses 2 spark plugs with screw fitted sleeve and 3 valves does not
infringe patent of the defendant, for permanent injunction restraining the
defendant from continuing the issuance of threats that the plaintiff is
infringing the defendant's patent directly or indirectly in any manner including
by way of circulars, advertisements, communications orally or in writing to
the plaintiff or any other person.

Issue:

Whether the Defendants have infringed the Plaintiff’s patent or combination


even though it made some improvements to the main patented article?

Judgment:

The court ruled that the novelty stated to have been achieved by the applicant
by way of patent, coupled with its enablement, as proved by putting the
product in the market and that has earned usage in large extent and both
novelty and enablement have been established by the applicant for the purpose
of granting the order of injunction in favour of the applicant/plaintiff.

Proposition:

 A final claim cannot be invalidated merely, because it gives more construction


than what was available earlier during the time of provisional specification. 
 It is important to bear in mind that in order to be patentable an improvement
on something known before or a combination of different matters already
known, should be something more than a mere workshop improvement; and
must independently satisfy the test of invention or an "inventive step". 

190
 For deciding the tests for variant, the English Court in Improver Corporation
and Ors. v. Remington Consumer Products Ltd. and Ors., gave three questions
which are as follows:

 The proper approach to the interpretation of patents registered under the


Patents Act 1949 was explained by Lord Diplock in Catnic Components Ltd.
v. Hill & Smith Ltd. The language should be given a “purposive” and not
necessarily a literal construction. If the issue was whether a feature embodied
in an alleged infringement which fell outside the primary, literal or a
contextual meaning of a descriptive word or phrase in the claim (“a variant”)
was nevertheless within its language as properly interpreted, the court should
ask itself the following three questions: (1) Does the variant have a material
effect upon the way the invention works? If yes;, the variant is outside the
claim. If no – (2) Would this (i.e. that the variant had no material effect) have
been obvious at the date of publication of the patent- to a reader skilled in the
art. If no, the variant is outside the claim. If yes – (3) Would the reader skilled
in the art nevertheless have understood from the language of the claim that the
patentee intended that strict compliance with the primary meaning was an
essential requirement of the invention. If yes, the variant is outside the claim.

 “It is also clear as per the decisions, for the purpose of deciding the novel
features to constitute “pith and marrow” a purposive construction has to be
given in order to make it essential requirement of the invention that any
variant would follow outside the monopoly even if it could not have material
effect upon the working of invention.”

H. PATENT LINKAGE
53. Bayer Corporation and Ors. v. Union of India and Others
Facts of the Case – Bayer are a foreign company incorporated under the
laws of Indiana, USA. Bayer had sought for a patent over an anti-kidney cancer
drug, ‘Nexavar’ which was granted in March 2008, providing for protection
until 2028. The Indian generic manufacturer had filed an application for the
license to manufacture, sell and distribute its drug, ‘Soranib’ which is a
substitute for the patented drug under the Bayer‘s patent. A writ petition was
filed by Bayer, to restrain the grant of the drug license by DCGI to sell,
manufacture and distribute the generic version of the drug.
Issues Raised – Bayer raised the following contentions;
(1) Firstly, the issue of patent linkage arising from challenge of DCGI‘s
marketing approval was raised.
(2) Secondly, it is contended that ‘Soranib’, generic drug is an imitation and
‘substitute’ of the patented drug and hence it falls within the definition of
‘spurious drugs’.
(3) Thirdly, the issue of the jurisdiction of the Drugs Controller General of
India was raised. It is not barred from considering the applications of other laws
as per Section 2 of the Drugs Act while granting marketing approval and hence
the grant of license to Cipla would be violative of Section 48 of the Patents Act.

191
Analysis of Issues
(1) The court examined the ‘Patent Linkage’ issue as the first point. The
issue should not be discussed in isolation considering the concept of ‘Patent
Linkage’. Peer review of both the legislations would be necessary to assess the
adversities of the linkage system, in the public health scenario, especially in
case of developing countries. The parliament has never expressed the intention
to link the drug approval procedures with enforcement of patents and interfacing
the powers of competent authorities under both legislations. The Drugs Act was
being a public regulatory measure and the Patents Act being right providing
legislations have different objectives and different authorities under both the
Acts. The court
(2) The Court examined the next issue raised that Cipla’s generic version,
‘Soranib’ is an imitation of or substitute for the patented drug. Section 17B of
the Drugs Act states that a spurious drug is not an imitation or a substitute of the
patented drug, but an imitation and substitute of another drug which resembles
and is likely to deceive the public. A generic drug is a copy which is the same as
a brand name drug in dosage, safety, strength and also with respect to the
quality, performance and intended use. The Drug Control Authority is
established for the grant of marketing approval to ‘generic drugs’ which was
highly misrepresented by Bayer as ‘spurious drugs’. The interpretation of
‘spurious as tanta mounting to generic’, would connect the counterfeit with
generic drugs, and the purpose of multinational companies to block the generic
entry into markets would be accomplished.
(3) It was averred that Chapter XVIII of the Patents Act dealt with suits
which are concerned with the infringement patetns, and Section 104 clearly
specifies that no suit relating to infringement shall be instituted in any court
inferior to the District Court. Thus, where the jurisdiction is specifically
mentioned then the alleged power of the DCGI would be ultra vires to Patents
Act and legislative intention of the Parliament.
Decision of the Court
In an attempt to caution against the demerits of patent linkage, the following
demerits of the system were pointed out;
a. It would vest new powers with authorities which are solely
concerned with scientific quality.
b. It would transform the patent rights which are private property
rights into public rights.
c. Such patent linkage also undermines the ‘bolar/early working’
exception which encourages quick access to the post patent
markets for generic medicines.
d. Art. 27 of TRIPS necessitates that patent should be made
available without any discrimination by the field of
technology.
In the concluding remarks the court observed that generic drugs would not
ipso facto amount to spurious drugs. The court held that there are mandatory

192
procedures and competent authorities for enforcing the patent infringement and
the plea of Bayer that the grant of approval by the DCGI would amount to an
infringement of patents is contrary to procedure established by law.

I. COMPULSORY LICENSING
54. Natco Pharma Ltd. vs. Bayer Corporation,[Before the Controller of
Patents, Mumbai]—12TH MARCH 2012
Article 30 and 31 of the TRIPS allows member countries to grant compulsory
licence to prevent the abuse of patent right. The WTO states that compulsory
licensing is when a government allows someone else to produce the patented
product or process without the consent of the patent owner.
The Patentee: M/s Bayer Corporation.
Product in issue: The product developed by the Patentee in 2005 and launched
under the trade name Nexaver for the treatment of Renal Cell Carcinoma-RCC
(Kidney cancer) and subsequently approved for treatment of Hepatocellular
Carcinoma-HCC (liver cancer) in 2007.
The Applicant: Natco Pharma who has developed the process to manufacture
the above mentioned drug and received a licence from the drug controller general
of India for manufacture of the drug in bulk.
Cost of the therapy offered by the Patentee: Rs. 2,80,428/- per month and Rs.
33,65,136/- per year.
The Application for compulsory licence was filed on 29.07.2011 under Section
84 (1) of the Patents Act, 1970 r/w Rule 96 of the Patent Rules, 2003.
Issues: [Section 84 (1) (a. b and c)]:
a) Whether the reasonable requirements of the public with respect to the
patented invention have not been satisfied?
The Patentee claimed that there are 8,842 cancer patients in India and that they
have sold 593 boxes in 2011. Here, the Patentee has made available the drug only
to a little above 2% of the eligible patents (1 patient requires 3 boxes of the drug).
According to the Applicant, the annual requirement in India is about 70,000 boxes.
Therefore, it was decided that the conduct of the patentee was unjustifiable.
b) Whether the patented invention is not available to the public at a reasonably
affordable price?
The patentee argued that the term ‘reasonably affordable price’ should be
construed with reference to the public as well as the patentee. The Controller did
not agree with this argument and held that ‘reasonably affordable price’ has to be
construed with reference to the public alone.
c) Whether the patented invention is not worked in the territory of India?
The Controller referred to the Paris Convention, TRIPS agreement and the
Patents Act and found that ‘working’ does not in any manner imply importation.

193
The Controller observed that ‘working’ means ‘manufacture to a reasonable extent
in India’. The Patentee has failed to comply with it.
Therefore, it was decided that the reasonable requirements of the public with
respect to the patented invention have not been satisfied and consequently a
compulsory licence be issued to the Applicant under Section 84 of the Act.

J. CLAIM CONSTRUCTION
55. Biosig Instruments Inc v. Nautilus Inc (US Court of Appeals for Federal
Circuit) Decided on April 26, 2013
Facts: Biosig is the assignee of U.S. Patent which is directed to a heart rate
monitor associated with an exercise apparatus and/or exercise procedures. It
filed a patent infringement suit against Nautilus. On the other hand Nautilus
filed a motion for summary judgment seeking to hold the patent in question
invalid for indefiniteness which was subsequently granted by the district court.
The term “spaced relationship” as recited in claim 1 was held to be indefinite as
no specific dimension or parameter of the space between two electrodes was
mentioned. Biosig appealed.

Question for consideration:


When can a claim be said to be invalid for indefiniteness?
Decision of CAFC:
A claim is indefinite when it is “insolubly ambiguous” or not “amenable to
construction”. A claim is “insolubly ambiguous” if reasonable efforts at claim
construction result in a definition that does not provide sufficient particularity
and clarity to inform person ordinary skilled in the art of the bounds of the claim
(metes and bounds of the claim cannot be ascertained).
General principles of claim construction apply when determining
indefiniteness, so claim language, specification and prosecution history are
determinative factors. The court can also have recourse to extrinsic evidence.
However, the court also clarified that it was not required that claims be plain
on their face in order to avoid a determination of invalidity for indefiniteness.
The court also rejected the contention the mere fact that there may be some
need for experimentation to determine the scope of the claims would render the
claims indefinite.
In the instant case, the CAFC held that the term “spaced relationship” does
not suffer from indefiniteness. Though no exact parameters were defined but the
court was of the view that patent’s claim language, specification, and the figures
illustrating the “spaced relationship” between the live and common electrodes
are telling and provide sufficient clarity to skilled artisans as to the bounds of
this disputed term. (It could be made out by PHOSITA from patent’s claim that

194
size between the two electrodes could not be larger than one’s hand or infinitely
small.) The CAFC reversed and remanded the district court’s decision.
Present Status: Certiorari has been granted by SC.

56. Scopema Sarl v. Scot Seat Direct Limited


Patents County Court [2013] EWPCC 32
Principle of claim construction- Although it is necessary to construe the
claims of a patent independently of any alleged infringement, it is a waste of
effort to contemplate the precise construction of all the integers which have
not been put in issue. Even those integers which are in issue may be difficult
to construe in the abstract; it is often only by looking at the alleged
infringement that one can appreciate the construction issues that actually arise.
The real point is that you should not allow the fact that the Defendant is doing
something in a particular way to influence you in deciding whether the claim
on its true construction covers that or not.

************************************

195

You might also like